Download as pdf or txt
Download as pdf or txt
You are on page 1of 113

ANKARA YILDIRIM BEYAZIT ÜNİVERSİTESİ TIP FAKÜLTESİ

2022 - 2023
PHASE TWO
COMMITTEE 4 GATHERED QUESTIONS
Anatomy
6. Which of the following is extension of internal
1. Which bony feature is a landmark for pudental oblique muscle of abdominal wall?
anesthesia? A. External spermatic fascia
A. Ischial tuberosity B. Cremasteric fascia
B. Obturator foramen C. Internal spermatic fascia
C. Iliac crest D. Dartos layer
D. Pubic tubercle E. Camper’s fascia
E. Ischial spine
7. Which of the following statements is false?
2. Left testicular vein drains into which vein? A. Right kidney is lower than left one
A. Left renal vein B. Ilioinguinal nerve is posterior to kidney
B. Inferior vena cava C. Right renal vein is shorter than left one
C. Internal iliac vein D. Left renal artery is shorter than left one
D. Prostatic venous plexus E. Efferent arteriol contributes to juxtaglomerular
E. Internal pudental vein apparatus

3. Which artery is not a branch of internal pudental 8. Which of the following is in the superficial
artery? perineal pouch?
A. Posterior scrotal artery A. Deep transverse perineal muscle
B. Helicine arteries B. Superficial transverse perineal muscle
C. Deep artery of penis C. Bulbourethral gland
D. Middle rectal artery D. Membranous part of urethra
E. Artery of bulb E. External urethral sphincter

4. Which of the following does not bound 9. Injury of which spinal cord segment leads
ischioanal fossa? erection problems?
A. Urogenital diafragm A. Col-2
B. Gluteus maximus muscle B. L1-2
C. Obturator externus muscle C. L2-3
D. Levator ani muscle D. S2-4
E. Sacrotuberous ligament E. L4-5

5. Which of the following is a peritoneal extension? 10. Which of the following statements is false?
A. Mackenrodt ligament A. Inferior angle of trigone is formed by internal
B. Cardinal ligament urethral orifice
C. Broad ligament B. Navicular fossa is a feature of male urethra
D. Round ligament C. Prostatic utricle is a feature of prostate
E. Uterosacral ligament D. Median umbilical ligament attaches to apex of
bladder
E. Internal urethral sphincter surrounds
preprostatic part of urethra
11. Cremasteric branch arises from which artery? 16. Which artery may be damaged due to its close
A. superficial epigastric artery proximity to transvers cervical
B. inferior epigastric artery ligament in uterine surgery?
C. external pudental artery A. External pudental artery
D. superior epigastric artery B. Internal pudental artery
E. internal pudental artery C. Middle rectal artery
D. Uterine artery
E. Ovarian artery
12. Which vessel is not crossed by ductus
deferens? 17. Which is not found in inguinal canal?
A. inferior vesical artery A. Spermatic cord in men
B. external iliac vein B. Round ligament in women
C. obturator artery C. Testicular artery
D. external iliac artery D. Ductus deferens
E. inferior epigastric artery E. Ovarian artery

13. Which of the followings is a modified sweat


gland? 18. Which structure is found in spermatic cord?
A. Scene's gland A. Femoral branch of genitofemoral nerve
B. Bulbourethral gland B. Iliohypogastric nerve
C. Mammary gland C. Inguinal branch of ilioinguinal nerve
D. Bartolin's gland D. Genital branch of genitofemoral nerve
E. Small vestibular glands E. Anterior scrotal branch of ilioinguinal nerve

14. Which structure prevents backflow of semen


into bladder during ejeculation?
A. utricle
B. internal urethral sphincter
C. urethral crest
D. navicular fossa
E. external urethral sphincter

15. Umbilical artery arises from which of the


followings?
A. obturator artery
B. external pudental artery
C. internal pudental artery
D. internal iliac artery
E. external iliac artery
C. Weight gain
Physiology D. Increased heart rate
1. A healthy 25-year-old woman was a subject in E. Poor memory
an approved research study. Her average urinary
urea excretion rate was 12 mg/min, measured over 6. Which one of the following conditions
24-hour period. Her average plasma urea and interventions is not expected to
concentration during the same period was 0.25 mg decrease the risk of Type 2 Diabetes
/mL.What is her calculated urea clearance ? mellitus? A. Increased diversity in the gut
microbiota
A. 0.25 mL/min B. Higher fibre intake
B. 288 mL/min C. Decreased short chain fatty acid production
C. 48 mL/min D. Improved postprandial glycemic control
E. Delay of sugar diffusion and absorption
D. 148 mL/min
E. 3 mL/min
7. Which of the following is false for the menstrual
cycle?
A. Progesterone inhibits myometrial contractions
2. In response to a physiological stimulus such as B. First 3-5 days of cycle which the menstrual
the stress of taking an important 4th committee flow occurs called menstrual phase
exam, which of the following reflects the most C. Progesterone causes the cervical mucus to
likely sequence of events? become thick and sticky
A.Corticotropin-releasing hormone, ^corticotropin, D. Endometrium thickens under the influence of
^cortisol estrogen.
B. ^Cortisol, ^corticotropin,(decrease)corticotropin- E. In proliferative phase, endometrial glands
releasing hormone become coiled and filled with glycogen.
C. Cortisol, ^corticotropin, ^corticotropin-releasing
hormone 8. A 54-year old woman received a life-saving
D. ^Corticotropin-releasing kidney transplant 6 months ago and had been
hormone,^corticotropin, (decrease)cortisol E. well until the past few days. She now reports
^Cortisol, (decrease)corticotropin, ^ severe fatigue and dizziness upon standing.
corticotropin-releasing hormone Urinalysis is positive for glucose, and there is
excessive excretion of HCO3 – and phosphate. In
3. Which of the following tissue is not dependent which segment of the nephron is function most
on insulin for glucose uptake? likely abnormal?
A. Adipose tissue A. Proximal tubule
B. Cardiac muscle B. Early distal tubule
C. The uterus C. Loop of Henle
D. The brain D. Collecting duct
E. Skeletal muscle E. Late distal tubule

4. Why do we pass more urine during the winter 9. Which is the correct order of these events that
and cold season? took place during and before fertilization?
A. Increased carbohydrate consumption
B. Increased secretion of ADH
I. Ciliary motions sweep the egg into the fallopian
C. Reduced sweating
tube.
D. Constriction of the afferent arterioles
II. Sperm reach the egg’s plasma membrane and
E. Decreased absorption of nephrons
fuses with this membrane.
5. Which of the following we would not expect to
see in a hypothyroid patient? A. Cold tolerance
B. Depressed mood
III. With the fusion of the pronuclei, a new cell , the D. Reabsorption of the solutes creates an
zygote, is formed. osmotic gradient that promotes the reabsorption of
water via osmosis.
IV. Acrosome reaction occurs. E. The high osmolarity of the interstitial fluid in
A. II – IV – I – III the renal medulla is due mainly to Na+, Cl-, and
B. I – III – II – IV urea.
C. I – IV – II – III
D. IV – I – II – III 14. Cortisone is administered to a 33year-old man
E. I – II – III – IV for the treatment of an autoimmune disease.
Which of the following is most likely to occur? A.
Increased muscle mass
10. Which of the following represents a
physiological action of growth hormone? A. B. Increased insulin secretion
Increases the breakdown of muscle protein C. Increased ACTH secretion
B. Increases utilization of glucose in muscle D. Increased cortisol secretion
C. Decreases storage of lipids in adipose cells E. Hypoglycemia between meals
D. Decreases gene transcription
15. A patient has severe vomiting.When you look at
E. Decreases gluconeogenesis in the liver
the arterial blood profile, you saw that both pH, [
HCO3-] (bicarbonate concentration) and pCO2 (
11. What is the primary event that induces
carbon dioxide partial pressure. has decreased.
ovulation?
What acid-base disorder does this patient have?
A. Low but adequate LH concentration
A. Respiratory alkalosis
B. LH peak
B. Respiratory acidosis
C. Inhibin hormone secretion
C. Metabolic alkalosis
D. Large quantities of progesterone concentration
D. Mixed acid-base disorder
E. Estrogen at low concentrations
E. Metabolic acidosis
12. Which of the following statements related with
16. Which one is a correct explanation for the
the Loop of Henle is not correct?
antiinflammatory effect of cortisol treatment?
A. It is located between proximal tubule and distal
A. Increased formation of leukotrienes
tubule.
B. Increased capillary membrane permeability
B. It aids in the urine’s concentration process.
C. Stabilization of cellular lysosomal membranes
C. Potassium sparing diuretics would be effective
in this nephron part. D. Increased release of pyrogen from granulocytes
D. It is an important place for the recovery of the E. Activation of phospholipase A2
sodium chloride.
17. Which of the following pair of elements & proteins
E. It allows the ions to pass freely from lumen to
is involved in the production of thyroid hormone?
intersititum in ascending loop.
A. Calcium-Albumin
13. Which of the following statements related with B. Magnesium - ThyroglobulinC. lodine -
the excretory system is not correct?
Hemoglobin
A. Lack of voluntary control over micturition is
termed urinary incontinence. D. Iron - Thyroglobulin
B. About 2150 mL of blood enters the renal E. lodine Thyroglobulin
arteries each minute.
18. Which statements is true of the fetal circulation?
C. Dilute urine is produced when the thick
A. The umbilical vein carries well-oxygenated blood.
ascending limb of the nephron loop, the distal
B. The foramen ovale typically closes by 32 weeks'
convoluted tubule, and the collecting duct reabsorb
gestation.
more solutes than water.
C. Ninety (90) percent of the cardiac output goes to
the lungs via the pulmonary
D. Blood shunts across the duct left to right' (from the 24. Which one of the following is not the function of
aorta to the pulmonary artery). the placenta? A. Secretes estrogen
E. The arterial duct (ductus arteriosus) helps send B. Secretes oxytocin during parturition. C.
oxygenated blood to the brain. Facilitates removal of carbon dioxide and
waste material from embryo.
19. Which one of the following organs and tissues is D. Secretes placental lactogens (hPL) and
insulin dependent? A. Macrophages placental growth hormone (hGH).
B. Kidney medulla E. Facilitates supply of oxygen and nutrients to
C. Skeletal muscle embryo.
D. Brain
E. Red blood cells
25. What is the mechanism by which the zona
pellucida becomes "hardened" after penetration of a
20. Which of these chemical buffering systems play
sperm cell to prevent a second sperm from
a role in the acid-base balance? I. Bicarbonate
buffer system penetrating?
II. Phosphate buffer system A. Decrease in estradiol levels
III. Protein buffer system B. An increase in intracellular calcium in the oocyte
A. I and II C. An increase in testosterone that affects the
B. II and III sperm
C. I and III D. The proteins released from the acrosome of the
D. Only I sperm
E. I, II and III E. Increased activity of the acrosomal enzymes
26. Which one of the following event sequences is
21. Which of the following hypothalamic nuclei is most not correct?
important for encoding the set point for daily circadian A. Chronic kidney disease - disturbed sodium
rhythms? A. Arcuate nucleus
and water balance - increased vascular
B. Paraventricular nucleus
C. Suprachiasmatic nucleus volume
D. Supraoptic nucleus B. Chronic kidney disease - disturbed
E. Preoptic anterior nucleus erythropoietin production - anemia
22. Which of the following statements about
calcium and phosphate balance in the body C. Chronic kidney disease - disturbed activation
is false? of vitamin (d) hypercalcemia
A. Calcitonin decreases plasma calcium D. Chronic kidney disease - disturbance in the
concentration eliminination of the nitrogenous wastes -
B. Vitamin D balances bone remodelingC.
Reduced plasma calcium stimulates uremia
parathyroid hormone secretion
E. Chronic kidney disease disturbed phosphate
D. Parathyroid hormone increases calcium
elimination - hypocalcemia
excretion from kidneys
E. Vitamin D stimulates calcium and phospate
27. A 37-year-old woman reports headaches and
absorption from the intestinal tract
frequent urination. Laboratory values reveal the
following information: Urine specific gravity =
23. Which of the following is not expected to be
1.003, urine protein = negative, plasma sodium =
handled by the reabsorption process more than
165 meq/l, plasma potassium = 4.4 meq/l, plasma
60% in the proximal tubule of the nephron?
creatinine = 1.4 mg/dl, blood pressure = 88/40 mm
A. Water
Hg, heart rate = 115 beat/min.
B. Glucose
What is the most likely cause of her elevated
C. HCO3-
plasma Na+ concentration?
D. D. Amino acids
E. Urea
A. Primary aldosteronism
B. Diabetes Mellitus D. Distinguishing sexual characteristics of external
C. Simple dehydration caused by insufficient water genitalia begin to appear during the ninth week
intake and exercise E. Feminization of the indifferent external genitalia
D. Diabetes insipidus occurs without the presence of a sex
E. Liddle Syndrome (an inherited form of high hormone
blood pressure)
4. Which one is a special staining for light
28. A 4-year-old boy was diagnosed with visual microscopy of Herring bodies which represent
disturbances due to a pituitary tumor secreting accumulations of neurosecretory granules?
excess growth hormone. Which of the following A. Mallory Azan
conditions would this boy most likely develop B. Trichrome Masson
without treatment? A. Dwarfism due to excess C. Periodic Acid (PAS)
IGF-1 production. D. Chrome-alumhematoxylin
B. Gigantism due to excess IGF-1 production C. E. Glial Fibrillary Acidic Protein (GFAP)
Dwarfism directly due to excess growth hormone
D. Acromegaly directly due to excess growth 5. Which cells are derived from the neural crest
hormone and a part of the DNES?
E. Gigantism directly due to excess growth A. Chromaffin cells of adrenal medulla
hormone. B. Pituicytes of neurohypophysis

Histology C. Oxyphil cells of parathyroid gland


D. Follicular cells of thyroid gland
1. Which of the following is not a unit of renal E. Pinealocytes of epiphysis
corpuscle? A. Juxtaglomerular cells
B. Mesangium
C. Glomerular capillaries 6.Which cells of pancreas have secretory granules
D. Parietal layer of Bowman’s capsule of 250-300-350 nm in their cytoplasm respectively
E. Visceral layer of Bowman’s capsule and truly identified by Mallory Azan method?
A. delta- brownish orange, beta-blue, alpha-red
2. Which of the following is not functions of
intraglomerular mesangium? B. alpha-red, beta-brownish orange, delta-blue
A.Controlling the turnover of the glomerular
basal lamina material by their phagocytic C. beta-red, delta-blue, alpha-brownish orange
activity.
B.Regulating blood flow by their contractile activity. D. alpha-brownish orange, delta-blue, beta-red
C. Maintenance of renal architecture and
production of erythropoietin. E. beta-brownish orange, alpha-red, delta-blue
D. Providing mechanical support for the glomerular
capillaries.
E. Secreting prostaglandins and endothelins. 7. Which of the following cells is responsible for
testosterone production?
3. Which statement is false for the development A. Granulosa cells
of external genitalia? B. Sertoli cells
A.The external genitalia are not fully differentiated C. Microglia
until the twelfth week D. Leydig cells
B. The urorectal septum fuses with the E. Macrophage
cloacal membrane at the end of the twelfth
week C. The external genitalia are sexually
undifferentiated from the fourth to the seventh
week 8. Which type of glial cells do the interstitial cells of
pineal gland resemble according to their B. An anterior vesicle part and a posterior
staining and ultrastructural features? phallic part
A. Oligodendrocytes C. An anterior pelvic part and a posterior vesicle
B. Schwann cells part
C. Pinealocytes D. An anterior urogenital sinus and a posterior
D. Microglia anorectal canal
E. Astrocytes E. An anterior anorectal canal and a posterior
urogenital sinus
9. Which one of the following information is wrong
for the mammary gland?
A. The inactive adult mammary gland is 13. Which organelle is abundant in Oxyphil cells
composed of 15 to 20 lobes separated by fibrous of the parathyroid gland and responsible for the
bands of connective tissue. strong acidophilia of these cells?
B. In the inactive gland, the glandular A. Golgi apparatus
component is consists of secretory units. B. Rough Endoplasmic Reticulum (RER)
C.Testosterone inhibit further growth of the C. Smooth Endoplasmic Reticulum (SER)
mammary gland. D. Mitochondria
D. The mammary glands in women undergo E. Lysosome
further development under hormonal influence of 14. Which of the following is not true for kidney?
estrogen and progesterone. A. Principal cell of collecting tubules has the
E. Mammary glands are modified tubuloalveolar receptors of aldosterone which stimulated the
apocrine sweat glands. reabsorbsion of Na+
10. Which of the following is the correct B. Mesangial cells participate indirectly in the
sequence in which urine flows through the kidney glomerular filtration process by providing
toward the urinary bladder? mechanical support for the glomerular capillaries
A. Ureter, minor calyx, major calyx, papillary C. The Fanconi syndrome is a renal hereditary
duct, renal pelvis disease in which distal convulated tubules fail to
B. Renal pelvis, major calyx, minor calyx, reabsorb amino acids and glucose.
papillary duct, ureter D. Cuboidal cells in distal convulated tubules
C. Papillary duct, minor calyx, major calyx, renal are shorter than those in the proximal convulated
pelvis, ureter tubules and lack a prominent brush boarder.
D. Minor calyx, major calyx, papillary duct, renal E. The macula densa, a region of the initial
pelvis, ureter portion of the distal convulated tubule and
E. Papillary duct, major calyx, minor calyx, sensitive to changes in NaCl concentration and
ureter, renal pelvis affects renin release by juxtaglomerular cells.

15. What is the epithelial type of the adult nipple


11. Which of the following cells is not found in the and areola?
seminiferous epithelium? A. Pseudostratified columnar epithelium
A. Leydig Cells B. Simple cuboidal epithelium
B. Sertoli cells C. Stratified cuboidal epithelium
C. Spermatogonia D. Keratinized stratified squamous epithelium
D. Spermatocytes E. Simple squamous epithelium
E. Spermatids
16. Which one of the following statements is false
12. What are the anterior and posterior cloacal for the development of pituitary gland?
partions divided by urorectal septum? A. The anterior lobe of the pituitary gland is
A. An anterior phallic part and a posterior pelvic derived from an evagination of the ectoderm of the
part oropharynx toward the brain
B. Pars tuberalis, develops from the thickened A. E: non-ciliated secretory columnar cells +
lateral walls of the Rathke's pouch and forms a ciliated cell LP: simple branched acinar glands B.
collar around the infundibulum E: non-ciliated secretory columnar cells +ciliated
C. Pars distalis, arises from the thickened cell LP: simple branched tubular glands C. E:
anterior wall of the Rathke's pouch and comprises non-ciliated peg cells + ciliated cell LP: simple
the bulk of the anterior lobe coiled tubular glands
D. Infundibulum is continuous with the median D. E: non-ciliated secretory columnar cells
eminence and contains the neurosecretory axons + ciliated cell LP: compound acinar glands
forming the hypothalamohypophysial tracts E. E. E: non-ciliated peg cells +ciliated cell
The posterior lobe of the pituitary gland is derived LP:
from a downgrowth of neuroectoderm of the compound tubular glands
floor of the fourth ventricle of the developing brain 21. Which are the followings are not the
components of the filtration barrier?
17. The decapitation process takes place in which A. Endothelium of glomerular capillary
of the following structures? A. Ductuli efferentes B. Pedicels
B. Ductus epididymis
C. Basal lamina
C. Prostate gland
D. Podocyte cell body
D. Ductus deferens
E. Nephrin protein
E. Seminal vesicles

18.Which is true for the development of thyroid


22. Which one is the most common cause of
gland?
female pseudohermaphroditism?
A. The thyroid gland begins to develop during the
fifth week of gestation A. Administration of androgenic agents during
B. About seventh week of gestation, epithelial cells pregnancy
of the fourth branchial pouches known as B. Ovarian tumors
ultimobranchial bodies start to migrate toward C. Masculinizing maternal tumors
the developing thyroid gland and cells give rise to D. Congenital Adrenal Hyperplasia
parafollicular cells. E. Testicular Feminization
C. The thyroid gland develops from the ectodermal
lining of the floor of the primitive
pharynx 23. In which stage of follicular development, a
D. About tenth week of gestation, ectodermal cells homogenous, deeply staining, acidophilic layer
differentiate into plates of follicular cells that called the zona pellucida appears between the
become arranged into follicles oocyte and the adjacent follicle cells and stromal
E. About fifteenth week of gestation, well developed cells immediately surrounding the follicle form a
follicles lined by the follicular cells contain colloid sheath of connective tissue cells, known as the
in their lumen theca folliculi? A. Primordial Follicle
B. Graafian Follicle
19. Which one of the following structures produce C. Antral Follicle
sperm? D. Secondary Follicle
A. Ejaculatory duct E. Primary Follicle
B. Seminal vesicles
C. Seminiferous tubules 24. Which one of the following statements is false
D. Prostate gland for zona fasciculate layer of adrenal cortex?
E. Bulbourethral glands A. The cells have spherical mitochondria with
tubular and vesicular cristae, and extensive
20. Which statement is true for the histology of networks of SER.
endometrium (epithelium, lamina propria)?
B. The polyhedral cells are smaller than the Silver and iodide reaction is negative, Acide
cells of the zona glomerulosa and are arranged in phosphatase positive, Autofluorescence negative
radial columns. E) In electron micrographs granules have
C.The sinusoidal capillaries arranged longitudinally eccentric electron dense core within the limiting
between the columns of cells. membrane of the granule, Azocarmin staning is
D.The cells have many lipid droplets in their obvious, Silver & iodide reaction is negative, Acide
cytoplasm and are called spongiocytes. phosphatase positive, Autofluorescence positive
E. The cells are large epithelioid cells and they
contain granules that stain lightly acidophilic. 28. Which of the following structures forms the
25. Which statement is false for vagina histology? main genital ducts in males?
A. Hymen, folds of mucous membrane are A. Paramesonephric ducts
derived from the ectodermal membrane in the B. Tunica albuginea
embryo B. vagina has few general sensory C. Neoral crest cells
nerve endings. C. The vagina possesses a D. Mesonephric ducts
stratified, squamous nonkeratinized epithelium E. The testis or medullary cords
and lacks glands
D. The vagina surface is lubricated mainly by mucus 29. Where do the primordial germ cells derived
produced by cervical glands E. The vagina is a from and when do they enter in the primary sex
fibromuscular tube cords during development of gonads?
A. Mesodermal cells of yolk sac, during the
26. In a ultrasonographic examination of a seventh week
pregnant woman, oligohydramnios and fetal B. Endodermal cells of yolk sac, during the
unilateral renal agenesis are detected. Which is seventh week
the most important cause of renal anomalies? A. C. Endodermal cells of yolk sac, during the fourth
Unilateral polycystic sendrome week
B. Oligohydramnios
D. Ektodermal cells of yolk sac, during the sixth
C. Absence of ureteric bud
week
D. Dejenaration of mesonephores
E. Endodermal cells of yolk sac, during the sixth
E. Ureter duplication
week

27. Which statement is true for the cells producing


30. What is the name of the artery in
Epinephrine in Adrenal Medulla? corticomedullary border in kidney? A. Dorsal
A) In electron micrographs granules are more aorta
homogenous &less dense than the cells producing B. Arcuate artery
Norepinephrine, Azocarmin staning is not obvious, C. Renal artery
Silver and iodide reaction is negative, Acide D. Intralobular artery
phosphatase negative, Autofluorescence positive E. Interlobular artery
B)In electron micrographs granules have eccentric
electron dense core within the limiting membrane of 2. Which of the following is true?
the granule, Azocarmin staning is not obvious, I. Leptin controls food consumption by
Silver and iodide reaction is positive, Acide suppressing neuropeptide Y synthesis.
phosphatase negative, Autofluorescence positive
II. Adiponectin decreases sensitivity to insulin,
C) In electron micrographs granules are more and its level increases in obese and diabetics.
homogenous &less dense than the cells producing
III. Progesterone increases blood glucose.IV.
Norepinephrine, Azocarmin staning is obvious,
Growth hormone activates hormone-sensitive
Silver and iodide reaction is positive, Acide
lipase in adipose tissue.
phosphatase positive, Autofluorescence negative
A. I and III
D) In electron micrographs granules are more
B. II and IV
homogenous &less dense than the cells producing
C. I,III and IV
Norepinephrine, Azocarmin staning is obvious,
D. I and IV E. Adrenalin
E. I,II and III
4. Which of the following is true regarding the
3. Which of the following hormones has the sources of hormones?
longest half-life? A. Glucagon I. Dihydrotestosterone originates from the
B. Insulin adrenal cortex.
C. Thyroxine II. Relaxin is a hormone derived from the ovary
D. ACTH and placenta.
6. Which of the following is true for hormones
Biochemistry and their receptors?

1. Which is not true for vitamin D metabolism? I. Receptors of natriuretic peptides use
A.1,25(OH)2D3 inhibits PTH synthesis in cGMP as a secondary messenger.
parathyroid gland II. Kinins act on B1 and B2 receptors
B. Calcitriol half life in circulation is shorter than increase following tissue damage.
half life of 25-hydroxy-VitD3 III. endothelin receptor is an intracellular
III. Estradiol is synthesized in the adrenal cortex, receptor.
adipose tissue, brain and Sertoli cells. IV. IV. ADH controls the excretion of water from the
Chronic gonadotropin is secreted from the kidneys via VI receptors.
placenta in pregnant women A. I and III
A. I and II B. I and IV
B. I and II C. II and III
C. II and III D. III and IV
D. II and IV E. I and II
E. I,II and IV

C. 1a-hydroxylation in kidney produces most


active form
D. Vitamin D receptor forms a dimer with the
retinoic acid X receptor on the nucleus
E. Hydroxylase enzymes in skin, liver and
kidney are responsible for the production of
metabolites of vitamin D

5. Which enzyme catalyzes the coupling 7. Which one is the secondary messenger
reactions in the thyroid gland for thyroid molecule that activates protein kinase A?
hormone synthesis? A. GMP
B. AMP
A. Oxidase C. cAMP
B. Reductase D. ATP
C. Peroxidase E. cGMP
D. Catalase
E. Dismutase 8. Which of the following is true?
I. Dihydrotestosterone binds minimally to
SHBG. II. Estriol is the best indicator of the
function of the fetoplacental unit. III.
Testosterone is converted to
dihydrotestosterone by 5-alpha reductase. IV.
Progesterone decreases cAMP by activating
phosphodiesterase ide the cell.
A. II,III, and IV B. Glucagon-like peptide
B. II and III C. Peptide YY
C. I and II D. Glucose-dependent insulinotropic
D. III and IV polypeptide
E. I,II and IV E. Glucagon
9. Which of the following is not true about water 13. Which of the following has no role in Renin
volume and osmo regulation in body? -
A. Changes in blood osmolarity is sensed by Angiotensinogen-Aldesterone system?
osmoreceptors result with changes in renal
water reabsorption A. Adrenal gland
B. ADH exerts its effects in proximal tubules B. Pineal gland
of kidney by promoting aquaporin protein C. Lung
movement and insertion into the apical D. Kidney
membrane C. Decrease renal perfusion in E. Liver
hypovolemia results with increase tubular Na+
and Cl- reabsorption through Angiotensin II
14. The relationship between obesity and
system D. Aldosterone increases transcription
insulin resistance has been described and
of sodium and potassium channels in the luminal
defined as the mechanism of metabolic
side of tubular lumen
syndrome. One of the following hormones
E. Hyperosmolar state stimulates Antidiuretic
…….I……. increases with the increased fat tissue
hormone (ADH) release
and causes insulin resistance; …….II.…..
increases when fat tissue decreases and
10. For diagnosis of adrenal medulla
insulin sensitivity by breaking insulin
chromaffin cell tumor, Pheochromocytoma,
resistance. In which case; fat tissue hormones
which one is measured in patient’s urine
that will come into spaces marked with I and II
sample?
is correctly given? A. I. Adiponectin II. Resistin
A. 5-OH Tryptophane
B. I. Resisten II. Adiponectin
B. Calcitriol
C. I. Resistin II. Leptin
C. Vanilmandelic acid
D. I. Ghrelin II. Leptin
D. Levodopa
E. I. Leptin II. Ghrelin
E. Monoamine oxidase

11. Which one of the hormones is derived from


15. Which of the following has only found in
preproghrelin protein and has inhibitory effect
Zona Glomerulosa of adrenal gland?
on food intake? A. Substance P
A. 21a-Hydroxylase
B. Obestatin
B. Aromatase
C. Motilin
C. Stetoridogenic acute regulatory (STAR)
D. Neurotensin
protein
E. Neuromedin U
D. 17a-Hydroxylase
E. 18 Hydroxysteroid dehydrogenase
12. Which one of the following hormones does
not
16. Which of the following hormones inhibit the
belong to secretin-family of
adenylate cyclase enzyme system?
gastrointestinal hormones?
A. ACTH
A. Vasoactive intestinal
B. Somatostatin
peptide
C. Calcitonin D. H20 is an acid and HF is the conjugate base
D. Glucagon E. HF is a base and F- is its conjugate acid
E. TSH
17. Which of the following statements is correct? 19. Which of the following membrane transport
A. Glucagon hormone consists of 2 amino acid proteins is responsible for transport of iodine
chains (a and ẞ chains), two disulfide bonds in thyroid tissue? A. lodide K+ channel
connecting the two chains, and a third disulfide B. VMAT
bond within the a chain. C. lonomycin
B. Resistin concentrations are low in obesity D. Aquaporin
and insulin resistance. E. Pendrin
C. Adiponectin accelerates the accumulation
of LDL in the arteries and increases the risk of 20. Which one of the following statements about
heart disease potassium metabolism is not correct?
D. After the leptin hormone is synthesized A. Hyperkalemia directly stimulates
from adipose tissue, it is released into the blood aldosterone release
and shows its effect in the hypothalamus. E. B. Potassium abnormalities might be caused
Ghrelin's task is to reduce appetite and to create by transcellular shifts
a feeling of satiety after the meal. C. Intracellular potassium is mainly found in
the muscle tissue
18. Which one of the following explains the D. Insulin increases potassium intake into
equation: HF+ H2O--> H3O++F- the cells
A. HF is a base and H30+ is its conjugate acid E. Alkalosis causes potassium to shift into
B. H2O is a base and HF is its conjugate acid the circulation
C. HF is an acid and F- is its conjugate base
21. Ali's grandmother has been suffering from
persistent vomiting for two days now. She appears
to be lethargic and weak and has myalgia. She is
diagnosed as having gastroenteritis & dehydration.
Measurement of arterial blood gas shows pH 7.5,
Pa02 85 mm Hg, PaCO2 40 mm Hg(35-45 mmHg),
and HCO3 34 mmol/L (24-26 mmol/L).
What acid- base disorder is shown?

A. Metabolic Alkalosis, Partially Compensated


B. Metabolic Alkalosis, Uncompensated
C. Respiratory Alkalosis, Uncompensated
D. Respiratory Alkalosis, Partially Compensated
E. Respiratory Acidosis, Partially Compensated

22. Which is a common feature of ANP and NO


receptors?
A. They have adenylate cyclase activity.
B. They activate gulanilate cyclase.
C. They have tyrosine kinase activity.
D. They are found in the cytosol.
E. They are located in the cell membrane.

23. Which of the following has 19 carbons?


A. Pregnenolone
B. Progesterone
C. DHEA
D. Cortisol
E. Cholestrol

24. Which of the following is/are expected in an


insulin dominated metabolism?
I. Phophoenolpyruvate carboxykinase is
active.
II. Acetyl CoA carboxylase is inactive.
III. Glycogen phosphorylase is active.
IV. Hormone sensitive lipase is active.
V. Phosphofrcutokinase is active.
A. IV & V
B. III & V
C. I & II
D. III & IV
E. II & III
Anatomy:
1.E 2.A 3.D 4.C 5.C 6.B 7.D 8.B 9.D 10.C

11.B 12.A 13.C 14.B 15.D 16.D 17.E 18.D

PHYSIOLOGY :
1.C 2.A 3.D 4.C 5.D 6.C 7.E 8.A 9.C 10.C

11.B 12.C 13.B 14.B 15.B 16.C 17.E 18.A 19.C 20.E

21.C 22.D 23.E 24.B 25.B 26.C 27.D 28.B

Histology :
1.A 2.C 3.B 4.C 5.A 6.B 7.D 8.E 9.B 10.C

11.A 12.D 13.D 14.C 15.D 16.E 17.B 18.A 19.C 20.B

21.B 22.D 23.A 24.B 25.A 26.C 27.D 28.D 29.E 30.B

Biochemistry :
1.E 2.C 3.C 4.E 5.C 6.E 7.C 8.B 9.B 10.C

11.B 12.C 13.B 14.B 15.E 16.B 17.D 18.C 19.E 20.E

21.A 22.B 23.C 24.B


‭ANATOMY‬

‭ - Which artery is not a branch of internal pudendal artery?‬


1
‭A)artery of bulb‬
‭B) posterior scrotal a.‬
‭C) helicine a.‬
‭D) deep artery of penis‬
‭E) middle rectal a.‬

‭2- Which artery does not supply the breast?‬

‭A) inferior epigastric‬ ‭B) axillary‬

‭C) internal thoracic‬ ‭D) lateral thoracic‬

‭ - Which of the following feature of the male genital system is external ?‬


3
‭A) Septi scrotum‬

‭ - Which of the following is false?‬


4
‭A) Ilioinguanal nerve is posterior the kidney‬
‭B) Effferent arteriol close to juxtoglomerular cells‬
‭C) Right vein is shorter than left one‬
‭D) Left artery is shorter than right one‬
‭E) Right kidney is higher than the left one‬

‭ - Which one is false?‬


5
‭A- Ilioinguinal nerve is posterior to kidney‬
‭B- Efferent arteriole contributes to juxtaglomerular apparatus‬
‭C- Right kidney is lower than left kidney‬
‭D- Right renal vein is shorter than left‬
‭E- Left renal artery is shorter than right‬

‭6- Which one of the followings is a peritoneal extention?‬

‭ ) Round Ligament‬
A ‭ ) Cardinal Ligament‬
B
‭C) Mackenrodt Ligament‬ ‭D) Broad Ligament‬
‭E) Uterosacral Ligament‬

‭ - Which features belong to deep inguinal ring?‬


7
‭A) Transversus abdominis muscle‬
‭B) External oblique muscle‬
‭C) External muscle aponeurosis‬
‭ - Deep inguinal ring sits on which structure?‬
8
‭A) Transversialis Muscle‬ ‭B) Transversial Fascia‬
‭C) Internal Oblique Muscle‬ ‭D) Peritoneum‬
‭E) External Oblique Muscle Aponeurosis‬

‭9- Which structure is in the inguinal ring?‬

‭10- The inguinal canal is located in:‬

‭ 1- Which of the following are does not pass near to ischioanal fossa?(something like that)‬
1
‭A) Levator ani muscle‬ ‭B) Urogenital diafgram‬
‭C) Oblique external muscle‬ ‭D) Gluteus maximus muscle‬
‭E) Sacrotuberous ligament‬

‭ 2- Which artery may be damaged due to its close proximity to transverse cervical ligament in uterine‬
1
‭surgery?‬
‭A) Internal pudendal artery‬ ‭B) Middle rectal artery‬
‭C) External pudendal artery‬ ‭D) Ovarian artery‬
‭E) Uterine artery‬

‭ 3- Which of the following can be damaged in surgery due to close proximity of supravaginal part of‬
1
‭the cervix?‬

‭A) Internal pudental‬ ‭B) Middle rectal‬

‭C) External pudental‬ ‭D) Uterine artery‬


‭E) Vaginal artery‬

‭14- Which of the following artery is located on the cortex-medullary line?‬

‭A)Arcuate Artery‬ ‭B)Interlobular Artery‬

‭C)Renal Artery‬ ‭D)Intralobular Artery‬


‭E)Interlobar Artery‬

‭15- Which one of the followings is a modified sweat gland?‬

‭A) Bartholin Gland‬ ‭B) Small Vestibular Gland‬

‭C) Bulbourethral Gland‬ ‭D) Scene’s Gland‬

‭E) Mammary Gland‬


‭16- Which one of the fallowings can't be seen in spermatic cord section ?‬

‭A) Testicular Vein‬ ‭B) Pampiniform Plexus‬

‭C) Genital br of Genitofemoral Nerve‬ ‭D) Vas Deferens‬

‭E) Femoral br of Genitofemoral Nerve‬

‭17- Which structure is a peritoneal fold?‬

‭A)Round ligament‬ ‭B) Transverse Cervical Ligament‬

‭C)Uterosacral ligament‬ ‭D) Suspensory Ligament of Ovary‬


‭E)Broad ligament‬

‭ 8- Which one is wrong about clitoris?‬


1
‭A) Erectile organ‬ ‭ ) Homologus to penis‬
B
‭C) Has 2 crura‬ ‭D) Has a body‬
‭E) It has glans‬

‭ 9- Which of the following is not true?‬


1
‭A) Prostatic u. is part of prostate‬
‭B) Navicular fossa is part of male urethra‬
‭C) Inferior angle of trigone marked by internal urethral orrifice.‬
‭D) Inferior spinchter cover preprostatic part of urethra‬
‭E) Medial umblical ligament extend from apex of bladder.‬

‭ 0- Which one is cortical structure of kidney?‬


2
‭A) major calyx‬ ‭B) renal pelvis‬ ‭C) vasa rectae‬ ‭D) glomerulus‬
‭BIOCHEMISTRY‬

‭ - In postmenaposual period, estrogen synthesis takes place in which of the following tissue from most‬
1
‭to least?‬
‭1- overs‬
‭2- Adrenal cortex‬
‭3- Adipose tissue‬
‭A-1,2,3‬
‭B-1,3,2‬
‭C-2,3,1‬
‭D-3,1,2‬
‭E-3,2,1‬

‭ - In the postmenopausal period estrogen synthesis take place in which of following from most to‬
2
‭least?‬
‭I-overs‬
‭II-adrenal cortex‬
‭III-peripheral tissues‬
‭A)I-II-III‬
‭B)I-III-II‬
‭C)II-III-I‬
‭D)III-I-II‬
‭E)III-II-I‬

‭ - Which of the following hormone are not used in congenital adrenal hyperplasia diagnosing?‬
3
‭A) Cortisol‬ ‭B) 21-deoxycortisol‬

‭C) 11-deoxycorsitol‬ ‭D) 17-hydroxyprogesterone‬


‭E) 11-hydroxyprogesterone‬

‭ - Which of the following actions are catalyzed by thyroperoxidase enzyme?‬


4
‭A) Oxidation of iodide‬
‭B) Incorporation of iodine to a thyrosine residue of thyroglobulin‬
‭C) Endocytosis of thyroglobulin‬
‭D) Deiodination of T4 in thyroid follicule‬
‭E) Condensation of monoiodothyrosine and diiodothyrosine cleavage and release of thyroid hormones‬

‭5- Which of the following does not suppress hunger‬

‭A) Ghrelin‬ ‭B) Pancreatic polypeptide‬ ‭C) Neuropeptide y‬

‭6- Which of the following hormone's receptor has enzymatic activitiy?‬

‭A) Cortisol‬ ‭B) Adrenaline‬ ‭C) Insulin‬ ‭D)Thyroxine‬ ‭E) Aldesteron‬


‭7- Which of the followings best describes ketoacidosis condition?‬

‭A) Metabolic acidosis with normal anion gap‬

‭B) Metabolic acidosis with high anion gap‬

‭C) Respiratory acidosis with normal anion gap‬

‭D) Respiratory acidosis with high anion GAP‬

‭E ) Respiratory alkalosis partially compansated with metabolic acidosis‬

‭8- Which cast can be visible in urine in physiological conditions?‬

‭A) Waxy Cast‬ ‭B) Granular Cast C) Hyaline Cast D) Leucocyte Cast E) Granular Cast‬

‭ - 30 year old female accepted to hospital with the symptoms,increased urine output,swelling in the‬
9
‭legs,weakness,nause,decreased glomerular filtration rate.Protein,Hb were positive in dipstick‬
‭test.Renal tubular epithelim is seen in urine.What could be possible diagnosis for this patient?‬
‭A) Acute Renal Failure‬

‭10- Which of the following hormon decrease when you gain weigt?‬

‭A)Leptin‬ ‭B)Rezistin‬ ‭C)Adiponectin‬ ‭D)Insulin‬ ‭E)Glukagon‬

‭ 1-) I-progesteron‬
1
‭II-testesterone‬
‭III-estradiol‬
‭Considering the number of carbon atoms of above hormones rank them largest to smallest‬
‭A)I-II-III‬ ‭B)I-III-II‬ ‭C)II-III-I‬ ‭D)III-I-II‬ ‭E)III-II-I‬

‭ 2- What is the mechanism of activation of alpha1 type adrenergic receptor?‬


1
‭A)Stimulation of calcium-calmodulin complex.‬
‭B)Inhibition of cAMP synthesis.‬
‭C)Activation of cAMP synthesis.‬
‭D)Stimulation of protein kinase-A activity.‬
‭E)Activation of G-alpha-stimulatory protein.‬

‭ 3- Which one is decrease ADH secretion?‬


1
‭A) Morphine‬ ‭B) Stress‬ ‭C) Nicotine‬ ‭D) Increased plasma osmolality‬ ‭E) Ethanol‬

‭ 4- Which hormone does not increase in stress?‬


1
‭A) Growth Hormone‬ ‭B) Cortisol‬ ‭C) Adrenaline‬ ‭D) Insuline‬ ‭E) Glucagon‬
‭ 5- From which of the following tissues does GH increase the release of insulin-like growth factors?‬
1
‭A) Pancreas‬ ‭B) Liver‬ ‭C) Hypothalamus‬ ‭D) Posterior pituitary‬ ‭E) Anterior pituitary‬

‭ 6- Which one of the following decrease in obesity‬


1
‭A) Leptin‬ ‭B) Adinopectin‬ ‭C) Insulin‬ ‭D) Glucagon‬

‭ 7- Which of the following factors does not Which of the following regarding fibroblast growth factor 23‬
1
‭(FGF23) effect on calcium phosphate metabolism is not true?‬

‭A) FGF23 decreases intestinal phosphate absorption and increases renal phosphate excretion.‬

‭B) FGF23 increases renal 1,25 (OH)2D synthesis by activating 1α-hydroxylase enzyme.‬

‭C) It is released by osteocytes.‬

‭D) 1,25 (OH)2D stimulates its release from cells.‬

‭E) Phosphate rich diets increase blood FGF23 levels.‬

‭ 8- Which of the followings are the members of POMC (proopiomelanocortin) peptide family?‬
1
‭1. Adrenocorticotropic hormone‬
‭2. TSH (I am not sure)‬
‭3. MSH‬
‭4. Beta-lipoprotein‬
‭A) 2, 3‬ ‭B) 1, 2, 3‬ ‭C) 1, 2, 4‬ ‭D) 1, 3, 4‬ ‭E) 2, 3, 4‬

‭ 9- Which of the following statement is correct for features of steroid hormones?‬


1
‭A) they are lipotrophic‬
‭B) act as a recopter hormone complex‬
‭C) they have a short life than protein containing hormone‬
‭D) their receptors are on the plasma membrane‬

‭ 0- Which hormon dont supress appetite?‬


2
‭A)Colesistokinin‬‭B)Ghrelin‬ ‭C)Pancreatic Polipetit‬ ‭D)GLP-1‬ ‭E)Peptide YY‬

‭ 1- Which of the following enzyme is responsible for the synthesis of epinephrine from‬
2
‭norepinephrine?‬

‭A) Amino acid decarboxylase‬ ‭B) Dopamine beta hydroxylase‬

‭C) Phenylethanolamine N-methytltransferase‬ ‭D) Thyrosine hydroxylase‬

‭E) DOPA decarboxylase‬


‭22- Brain natriuretic peptide (BNP) is important biomarker for chronic failure of which organ?‬

‭A)Liver‬ ‭B)Heart‬ ‭C)Pancreas‬ ‭D)Adrenal cortex‬ ‭E)Eye‬

‭23- Which of the followings gastointestinal-hormon function is wrong?‬

‭A) Glucagon like peptide 1 inhibits glucagon secretion‬

‭B) Gastrin stimulates pancreatic enzymes‬

‭C) Secretin stimulates insulin secretion‬

‭D)VIP stimulate water and electrolyte secretion‬

‭E) GIP inhibits lipoprotein lipase and fatty acid uptake‬

‭ 4- A patient came to the emergency service for parathyroid adenoma. For primary‬
2
‭hyperparathyroidism what could be find?‬
‭1) Increased PTH‬
‭2) Increased Calcitriol‬
‭3) Increased Serum Calcium‬
‭4) Decreased Bone Mineral Density‬
‭5) Increased Serum Phosphorus‬

‭A) 1,3,4,5‬ ‭B) 1,2,3,4‬ ‭C) 1,3,4‬‭D) 3,4,5‬‭E) 2,4,5‬

‭ 5- A patient with parathyroid adenoma. In this primary hyperparathyroidism patient which ones can‬
2
‭occur?‬
‭I.Increased PTH‬
‭II.Increased serum calcium‬
‭III.Increased bone calcitriol‬
‭IV.Decreased bone mineralization‬
‭V.increased serum phosphate‬

‭ 6- Which are the hormones That Use the Adenylyl Cyclase–cAMP Second Messenger System?‬
2
‭1-Somatostatin‬
‭2-Insulin‬
‭3-Glucagon‬
‭4-Tsh‬

‭A) 1‬ ‭B) 2-3‬ ‭C)2-4‬ ‭D)2-3-4‬ ‭E)1-3-4‬

‭27- Which does not belong to the proopiomelanocortin family?‬

‭A) B-lipotropin‬ ‭B) Oxytocin‬ ‭C) Endorphin‬ ‭D) MSH‬ ‭E) ACTH‬
‭28- Which of the gastrointestinal hormone-function pairings is wrong?‬

‭A) gastrin- stimulates pancreatic secretions‬

‭B) secretin- stimulates insulin secretion‬

‭ 9- Which one of the following is the most correct option regarding the characteristics of steroid‬
2
‭hormones?‬
‭I. They are lipophilic‬
‭II. Act as receptor enzyme complex‬
‭III. Their half-life is shorter than protein containing hormones‬
‭IV. Receptors are on the cell surface‬

‭A) I‬ ‭B) I-II‬ ‭C) II-III‬ ‭D) I-II-III‬ ‭E) I-III-IV‬

‭30- Features of subclinical hyperthyroidism?‬

‭A) high TSH and normal level free T3 T4‬ ‭B) high TSH and low level free T3 T4‬

‭C) normal TSH, low level free T3 T4‬ ‭D) low TSH, normal level free T3 T4‬

‭E) low TSH, high level free T3 T4‬

‭ 1- Which one of the following options are the hormones secreted with neurophysin 1, neurophysin 2‬
3
‭respectively?‬

‭A) Oxytocin, Vasopressin‬ ‭B) Vasopressin, Insulin‬

‭C) Vasopressin, Oxytocin‬ ‭D) TSH, Insulin‬


‭E) ACTH, Oxytocin‬

‭ 2- 10 year old boy develops diarrhea while travelling to India. What is the correct diagnosis for this‬
3
‭child ?‬
‭Arterial ph:7.25, pCO2: 24 mmHg, HCO3: 10 mEq/L‬

‭ . Metabolic acidosis‬
A ‭ . Metabolic alklosis‬
B
‭C. Respiratory alkalosis‬ ‭D. Respiratory acidosis‬
‭E. Mixed acid base disorder‬

‭ 3- What is the action mechanism of a1adrenergic reseptor?‬


3
‭A) It stimulate calcium calmudolin‬ ‭ ) It decrease cAMP‬
B
‭C) It increase cAMP‬ ‭D) It stimulate protein kinaz A‬
‭E) Ga stimulator protein‬
‭34- steroid ve protein hormonların farklarıyla ilgili bir soruydu‬

‭ 5- biyokimya labında daha önce isteğe bağlı 4 tane kan tahlilinin 2 si soruldu(falza su içen kadın ve‬
3
‭yorulmuş kadın) teorik sınavda soruldu‬

‭36- which transporter responsible for iodine into thyroid cell against concentration gradient?‬

‭ 7- 25 yaşında kadın hasta, şikayetleri var. Hastanede yatıyo 6 saat başka sıvı almadan ADH agonisti‬
3
‭veriliyor semptomları düzeliyor. Hastalığı?‬
‭A. Central diabetes insipidus‬
‭B. Nehprogenic diabetes insipidus‬
‭C. Ketoacidosis‬
‭D Diabetus mellitus type 1‬
‭E. Diabetes mellitus type 2‬

‭ 8- Which of the following is not expected after a large hemorrhage?‬


3
‭A. Increased ADH secretion‬
‭B. Increased sympathetic discharge‬
‭C. Increased aldosterone secretion‬
‭D. Decreased urine flow‬
‭E. Increased osmolar excretion‬

‭ 9- Which of the following statements is correct?‬


3
‭a) Glucagon hormone consist of 2 aminco acid chains(alfa and beta) two sulfide bonds connecting the‬
‭two cahins and a third bonds within the alfa chain.‬
‭b)Ghrelin's task is to reduce appetite and to create a fe...(?) after meal‬
‭c)Resistin concentrations are low in obesity and ins...(?)‬
‭d)After a leptine hormone is syntesis from adipse into the blood and shows its effect on hypothalamus.‬
‭e) Adiponectin declereates the accumulation increases the risk of head disease.‬

‭ 0- What nuclei synthesized melatonin‬


4
‭A)paraventreicular‬
‭B)arcuate‬
‭C)supraoptic‬
‭D)suprachiasmatic‬
‭E)paratohiasmatic‬
‭PHYSIOLOGY‬

‭ - Radiation treatment for a pituitary tumor in an 8-year-old boy resulted in the‬


1
‭complete loss of pituitary function. Which one of the following is he going to‬
‭experience as a result?‬
‭A. Increased ACTH‬
‭B. Hypothyroidism and goiter‬
‭C. Accelerated growth spurts‬
‭D. Increased TSH levels‬
‭E. Absent sexual maturation‬

‭ - Which one occurs in the proximal convoluted tubule?‬


2
‭A) Na-K symporter‬
‭B) Na-Ca symporter‬
‭C) Na-Cl symporter‬
‭D)Na-Hco3 symporter‬
‭E) Endothelial Na+ transporter?‬

‭ - Which one of the following factors does not cause a shift of potassium into cells?‬
3
‭A. Very heavy exercise‬
‭B. Insulin‬
‭C. Beta-adrenergic stimulation‬
‭D. Alkalosis‬
‭E. Aldosterone‬

‭ - Which one of the hormones has long term effect in response to stress?‬
4
‭A)Mineralocorticoid‬
‭B)Glucocorticoid‬
‭C)Insulin‬
‭D)Glucagon‬
‭E)Catecholamine‬

‭ - What is the class of adrenal gland hormones that provides resistance to stress,‬
5
‭produce anti-inflammatory effects, and promote normal metabolism to ensure‬
‭adequate quantities of ATP?‬
‭A. Androgens‬
‭B. Gonadocorticoids‬
‭C. Mineralocorticoids‬
‭D. Glucocorticoids‬
‭E. Catecholamines‬
‭ - Which one is not a characteristic effect of testosterone?‬
6
‭A) Increased muscle mass‬
‭B) Sex organ grow and function‬
‭C) Voice deepen‬
‭D) Broadening of pelvis‬
‭E) Hair growth on face and body‬

‭ - Which one of the following statements regarding glomerular filtration rate (GFR) is‬
7
‭correct?‬
‭A. Increased glomerular capillary hydrostatic pressure decreases GFR‬
‭B. Increased glomerular capillary colloid osmotic pressure increases GFR‬
‭C. Increased glomerular capillary filtration coefficiency decreases GFR‬
‭D. Increased Bowman’s Capsule hydrostatic pressure increases GFR‬
‭E. Any kind of obstruction of the urinary tract decreases GFR‬

‭8- Most people sleep at night and wake up on morning. What is the mechanism behind it?‬

‭A) Brain gets information about day&night from eyes and circadian rhythm synchronizes with it‬

‭ - Which of the following does not contribute to the formation of maximally‬


9
‭concentrated urine?‬
‭A. Presence of urea in the inner medullary interstitium‬
‭B. Active NaCl transport in the proximal convoluted tubule‬
‭C. Impermeability of the thick ascending limb of the loop of Henle to water‬
‭D. High water permeability of the collecting duct due to the presence of ADH‬
‭E. Active NaCl transport in the thick ascending limb of the loop of Henle‬

‭ 0- which one of the hormones cause rapid weight gain in face, chest, abdomen, high blood pressure,‬
1
‭osteoporasis depression when it is too much?‬

‭A)melatonin‬

‭B)estrogen‬

‭C)testosterone‬

‭D)cortisol‬

‭E)insulin‬

‭ 1- Which hormone stimulates weight and fat gain from chest, abdomen; its excess amount will cause‬
1
‭irritability, anxiety sometimes?‬

‭A) estrogen‬
‭ 2- Insulin and thyroxine arrive at an organ at the same time. Thyroxine causes an‬
1
‭effect on the organ but insulin does not. What could be the reason for this event?‬
‭A. Insulin is a lipid-soluble hormone and thyroxine is not‬
‭B. The organ’s cells have receptors for thyroxine but not for insulin‬
‭C. The target cells in the organ have up-regulated for thyroxine‬
‭D. Thyroxine inhibits the action of insulin‬
‭E. Thyroxine is a local hormone and insulin is a circulating hormone‬

‭ 3- Which of the following must occur for a pregnant woman to maintain a healthy‬
1
‭pregnancy?‬
‭A. The pituitary gland must secrete prolactin to sustain the placenta‬
‭B. The pituitary gland must secrete hCG to maintain the corpus luteum‬
‭C. The placenta must secrete FSH to maintain ovarian function‬
‭D. The placenta must secrete LH to maintain ovarian function‬
‭E. The corpus luteum must secrete progesterone to sustain the endometrium‬

‭ 4- What is the normal human glomerular filtration rate (GFR) (mL/min)‬


1
‭approximately?‬
‭A. 75‬
‭B. 125‬
‭C. 25‬
‭D. 50‬
‭E. 225‬

‭ 5- Which of the following hypothalemic nuclei is most important for encoding in the set point for daily‬
1
‭circadian rhythms?‬
‭A) arcuate nucleus‬
‭B)Supraoptic nucleus‬
‭C)Preoptic anterior nucleus‬
‭D)suprachiasmatic nucleus‬
‭E)paraventricular nucleus‬

‭ 6- Which of the following increase serum P levels?‬


1
‭A) Aldosterone‬
‭B) P shift to intracellular fluid‬
‭C) Acidosis‬
‭D) Increased tubular flow rate‬
‭E) Insulin‬
‭ 7- Which one of the following conditions cannot be considered among the causes of‬
1
‭prerenal acute kidney injury?‬
‭A. Malignant hypertension‬
‭B. Hemorrhage (trauma, surgery, postpartum, gastrointestinal)‬
‭C. Renal artery stenosis, embolism, or thrombosis of the renal artery or vein‬
‭D. Diarrhea or vomiting‬
‭E. Cardiac Failure, Myocardial infarction, or Valvular damage‬

‭ 8- Which statement is wrong about hormones?‬


1
‭a- growth hormone and thyroxine hormone are essential for development of body‬
‭b- estrogen ..?‬
‭c- ACTH stimulates release of a glucocorticoid hormone Cortisol from the adrenal glands‬
‭e- Dopamine stimulates Prolactin secretion and stimulates mammary glands to produce milk *‬

‭ 9- A 54-year-old woman received a life-saving kidney transplant 6 months ago and had‬
1
‭been well until the past few days. She now reports severe fatigue and dizziness upon‬
‭standing. Urinalysis is positive for glucose, and there is excessive excretion of HCO3‬
‭− and phosphate. In which segment of the nephron is the function most likely to be‬
‭abnormal?‬
‭A. Early distal tubule‬
‭B. Collecting duct‬
‭C. Proximal tubule‬
‭D. Late distal tubule‬
‭E. Loop of Henle‬

‭ 0- Regarding the relation between obesity and breast cancer which of the followings is not correct?‬
2
‭A)Early mensturation and late menapasue increases breast cancer‬
‭B)Obesity related hormone receptor increases breast cancer(not sure about this)‬
‭C)Obese women poor breast cancer prognosis‬
‭D)Most breast cancer occurs after menapause when the estrogen levels are low‬
‭E) after menopause main hormone responsible for breast cancer is progesterone severe from fat cells‬

‭ 1- Which one of the following effects is not observed by the actions of estrogen?‬
2
‭A) Inhibit osteoblastic activity in the bones‬
‭B) Cause cellular proliferation and growth of the tissues of the sex organs‬
‭C) Increase body metabolism and fat deposition‬
‭D) Promote the preservation of a homeostatic (eubiotic) vaginal microenvironment and‬
‭prevents infections‬
‭E) Cause the skin to become more vascular‬
‭ 2- Why do you think that a person consuming high protein will have increased urine concantrating‬
2
‭ability‬
‭A)high urea production helps hyperosmotic medullary interstitium‬
‭B)increased glomerular filtration rate‬
‭C)provides much more energy source for the kidney function‬
‭D)high structural protein formation in the kidney tissue‬
‭E)the improved tubular function just because of high protein formation‬

‭ 3- Which one of the following can not be regarded among the causes of insulin resistance?‬
2
‭A.Obesity/overweight(especially excess visceral adiposity)‬
‭B. Pregnancy, gestational diabetes‬
‭C.Lipodystrophy(acquired or genetic;associated with lipid accumulation in the liver)‬
‭D. Deficiency of growth hormone‬
‭E. Autoantibodies to the insulin receptor‬

‭ 4- Which one of the following nephron segments and function pairs is correct in respect of their‬
2
‭functional relationship?‬
‭A) Proximal Tubule - Water is absorbed into the medulla‬
‭B) Thick Ascending Loop of Henle - Osmolarity of the fluid depends on the level of ANP‬
‭C) Descending Loop of Henle - About 2/3 of the electrolytes and water are reabsorbed‬
‭D) Early Distal Tubule - Most diluted place of the filtrate normally‬
‭E) Late DT and C. Collecting Tubules - Active electrolyte transport into the medullary interstitium‬

‭ 5- Which one of the following statements about insulin action on different organs and tissues is not‬
2
‭true?‬
‭A) Descreased lipolysis in adipose tissue‬
‭B) Increased inflammation in adipose tissue‬
‭C) Increased muscle mass in skeletal muscles‬
‭D) Decreased glycogen synthesis in hepatocytes‬
‭E) Decreased fatty acid transport from blood‬

‭ 6- The syndrome of inappropriate antidiuretic hormone secretion (SIADH) is caused‬


2
‭by the excess release of ADH. Which one of the following is increased because of‬
‭SIADH?‬
‭A. Plasma osmolality‬
‭B. Plasma oncotic pressure‬
‭C. Intracellular volume‬
‭D. Urinary flow‬
‭E. The concentration of plasma sodium‬

‭ 7- Which of the following is wrong for the luteal phase?‬


2
‭A)GnRH stimulate the release of FSH and LH‬
‭B)Progesterone is secreted from the Corpus luteum‬
‭C)Progesterone provides the endometrium to become thicker‬
‭D)After ovulation the follicle filled with a fluid‬
‭28- Which is wrong for luteal phase?‬

‭A- corpus luteum secretes increasing amounts of ostrogen and progesterone.‬

‭29- What occurs when afferent arteriole has smaller diameter than efferent arteriole in gromeli?‬

‭ ) slight GFR increase‬


A
‭B) Increase pressure in glomerular ‬

‭ 0- Inhibition of ACTH by cortisol feedback type?‬


3
‭A)Ultra -short loop feedback‬
‭B)Short loop feedback‬
‭C)Indirect long loop feedback‬
‭D)Direct long loop feedback‬
‭E)Indirect ultra long loop feedback‬

‭ 1- Kid borns with a mutation that makes ascending henle permeable to water. Obligatory urine‬
3
‭concentrations?‬

‭ -100/?‬
A
‭B-100/?‬
‭C-300/?‬
‭D-300/?‬
‭E-1200/1200‬

‭ 2- What is the main source of ammonia produced by the kidneys?‬


3
‭A. Alanine‬
‭B. Leucine‬
‭C. Tryptophan‬
‭D. Glutamine‬
‭E. Glycine‬

‭ 3- Which one is used for production and secretion ammonium by proximal tubule‬
3
‭buffering of excess acid?‬
‭A) Glutamine‬
‭B) Histamine‬
‭C) Metionin‬
‭D) Tryptophan‬
‭E) Alanine‬
‭HISTOLOGY‬

‭ - At which stage of follicular development a deeply staining acidophilic layer called zona pellucida‬
1
‭appears around the oocyte and a sheet of connective tissue cells known as Theca Follicle surrounds‬
‭the follicle?‬
‭A. Primordial follicle‬
‭B. Secondary follicle‬
‭C. Graafian follicle‬
‭D. Antral follicle‬
‭E. Primary follicle‬

‭ - In pregnant woman's ultrasonography in there are oligohydramnios and fetal‬


2
‭unilateral renal agenesis detected. What is the most important cause of renal anomaly?‬
‭A) No ureteric bud formation (ya da‬‭Absence of ureteric‬‭bud tam hatırlamıyorum)‬
‭B) Unilateral polycystic syndrome‬
‭C) Ureter duplication‬
‭D) Oligohydramnios‬
‭E) Degeneration of mesonephros‬

‭ - Which of the followings is not a unite of renal corpuscle? *‬


3
‭A)Visceral layer of Bowman Capsule‬
‭B)Parietal Layer of Bowman Capsule‬
‭C)Mesangium‬
‭D)Juxtaglomerular Cell‬
‭E)Glomerular Capillary‬

‭ - Which one of the following is false for beta cells of Langerhans islets?‬
4
‭A. They stain red with the Mallory Azan method‬
‭B. They constitute about 70% of total islet cells‬
‭C. They contain numerous secretory granules about 300 nm in diameter and secrete insulin‬
‭D. They have a dense polyhedral core and a pale matrix‬
‭E. They are generally located in the central portion of islets‬

‭ - Which of the following is not true for kidneys?‬


5
‭A. Mesangial cells participate indirectly in the glomerular filtration process by providing‬
‭mechanical support for the glomerular capillaries‬
‭B. The macula densa, a region of the initial portion of the distal convoluted tubule and‬
‭sensitive to changes in NaCl concentration and affects renin release by‬
‭juxtaglomerular cells‬
‭C. Principal cell of collecting tubules has the receptors of aldosterone which stimulated‬
‭the reabsorption of Na+‬
‭D. Cuboidal cells in distal convoluted tubules are shorter than those in the proximal‬
‭convoluted tubules and lack a prominent brush border‬
‭E. The Fanconi syndrome is a renal hereditary disease in which distal convoluted‬
‭tubules fail to reabsorb amino acids and glucose‬
‭ - Which layer of the adrenal gland develop in the early years of life and constitute the‬
6
‭darkly staining acidophilic cells that are ranged in anastomosing cords?‬
‭A) Adrenal capsule‬
‭B) Zona glomerulosa‬
‭C) Zona fasiculata‬
‭D) Zona reticularis‬
‭E) Adrenal medulla‬

‭ - In female reproductive system which organ do not differentiate from paramesonephric duct?‬
7
‭A) uterine tubes‬
‭B) uterus‬
‭C) cervix‬
‭D) upper portion of vagina‬
‭E) vestibule‬

‭ - Which one of the following is wrong for development of parathyroid gland?‬


8
‭A) They develop from endodermal cells derived from third and fourth branchial pouches.‬
‭B) Inferior parathyroid glands and thymus develop from third branchial pouch, superior parathyroid‬
‭glands develop from fourth branchial pouch.‬
‭C) The principal (chief) cells differentiate during embryonic development.‬
‭D) The oxyphil cells differentiate later at puberty.‬
‭E) Ultimobranchial bodies migrate towards parathyroid glands and give rise to oxyphil cells.‬

‭ - Which is the correct sequence in which urinary flows through the kidney towards to do urinary‬
9
‭bladder?‬
‭A) Minor calyx, major calyx, papillary duct, renal pelvis, ureter.‬
‭B)Minor calyx, major calyx, renal pelvis, papillary duct, ureter.‬
‭C)Major calyx, minor calyx, renal pelvis, papillary duct, ureter.‬
‭D)Papillary duct, minor calyx, major calyx, renal pelvis, ureter.‬
‭E)Ureter, major calyx, renal pelvis, papillary duct, minor calyx.‬

‭ 0- What is the way of urine from kidney to Ureter respectively?‬


1
‭A) renal papilla-minor calyx-major calyx-renal pelvis‬
‭B) renal pelvia-major calyx-minor calyx-renal papilla‬
‭C) renal papilla-major calyx-minor calyx-renal pelvis‬
‭D) renal papilla-renal pelvis-minor calyx-major calyx‬

‭ 1- What are the anterior and posterior cloacal portions divided by urogenital septum?‬
1
‭A) An Anterior Urogenital Sinus and a Posterior Anorectal Canal‬
‭B) An Anterior Anorectal Canal and a Posterior Urogenital Sinus‬
‭C) An Anterior Pelvic Part and a Posterior Vesical Part‬
‭D) An Anterior Phallic Part and a Posterior Pelvic Part‬
‭E) An Anterior Vesical Part and a Posterior Phallic Part‬
‭ 2- Which one of the following cells seen at the 6 years old boy testicular biopsy? histo‬
1
‭A) Spermatid‬
‭B) Spermatozoon‬
‭C) Myeloid cells‬
‭D) Secondary spermatocyte‬
‭E) Primary spermatocyte‬

‭ 3- Which one of the followings isn't originated from Mesonephric Duct?‬


1
‭A) Efferent Ductules‬
‭B) Epididymis‬
‭C) Seminal Vesicle‬
‭D) Vas Deferens‬
‭E) Cowper's Gland‬

‭ 4- Which one of the following is false for Thyroid follicles?‬


1
‭A. Follicles exhibit slightly basophilic cytoplasm with spherical nuclei containing one or‬
‭more prominent nucleoli‬
‭B. In routine H&E preparations, they are pale staining and occur as solitary cells or‬
‭small cluster of cells‬
‭C. Follicles reveal organelles commonly associated with both secretory and absorptive‬
‭cells (short microvilli on the apical surface) (numerous RER)‬
‭D. Follicles contain abundant lysosomes and endocytic vesicles identified as colloidal‬
‭resorption droplets‬
‭E. Follicles are responsible for the production of the thyroid hormones T4 and T3‬

‭ 5- Which one of the following statements is false for the development of the female‬
1
‭reproductive system?‬
‭A. Development of the sex-specific internal ducts and external genitalia defined during‬
‭fetal development‬
‭B. Gonad differentiation begins at the 7th week of gestation‬
‭C. Chromosomal sex is determined at conception‬
‭D. Secondary sex characteristics defined at puberty‬
‭E. Female genital organ development is dependent on hormones and inhibitory‬
‭peptides‬

‭ 6- Which histological layers of the endometrium constitute the functional layer and disintegrate and‬
1
‭shed at menstruration?‬
‭A) compact layer and basal layer‬
‭B) spongy layer and basal layer‬
‭C) compact layer and spongy layer‬
‭D) basal layer, spongy layer and compact layer‬
‭E) basal layer and myometrium‬
‭ 7- Which functional layers is shed when pregnancy doesn’t occur or supply uterus when pregnancy‬
1
‭occurs?‬
‭A) Compact layer and basal membrane‬
‭B) Spongy layer and basal membrane‬
‭C) Compact layer and spongy layer‬
‭D) Compact, spongy, basal membrane‬

‭ 8- Which of the following constitue approximately 15-20% of the parenchymal cells of the anterior‬
1
‭lobe and have polygonal medium-sized cells with round and eccentric nuclei and stain basophilic?‬
‭A) Somatotrops‬ ‭B) Corticotrops‬ ‭C) Lactotrops‬ ‭D) Gonadotrops E)Thyrotrops‬

‭ 9- Which cells are slightly basophilic have spherical nuclei with a single prominent nucleolus and‬
1
‭have small secretory vesicles producing melatonin?‬
‭A) Interstitial cells‬
‭B) pituicytes‬
‭C) oxyphil cells‬
‭D) pinealaocytes‬
‭E) clear cells (c-cells)‬

‭ 0- Which part of the nephron connects the Bowman's capsule to the descending loop?‬
2
‭A)Proximal convoluted tubule‬
‭B) Loop of Henle‬
‭C) Distal convoluted tubule‬
‭D) Glomerulus‬
‭E) Collecting tubule‬

‭ 1- What is the structure in between when going from Bowman's capsule to Henle's tubule?‬
2
‭A) proximal tubule‬
‭B) distal tubule‬
‭C) collecting duct‬

‭ 2- Which one of the following statements is false for the zona fasciculata layer of the‬
2
‭adrenal cortex?‬
‭A. The zone contains sinusoidal capillaries arranged longitudinally between the columns‬
‭of parenchymal cells‬
‭B. The cells have spherical mitochondria with tubular and vesicular cristae, extensive‬
‭networks of SER‬
‭C. The cells are large epithelioid cells and they contain granules that stain intensely with‬
‭PAS reagent‬
‭D. The cells have many lipid droplets in their cytoplasm and are called spongiocytes‬
‭E. The polyhedral cells are larger than the cells of the zona glomerulosa and are‬
‭arranged in radial columns‬
‭ 3- Which one is false for hypothalamus histology?‬
2
‭A) The hypothalamus is located in the middle of the base of the brain, and it encapsulated the central‬
‭part of the third ventricle.‬
‭B) It coordinates most endocrine functions of the body.‬
‭C) It serves as one of the major controlling centers of the autonomic nervous system.‬
‭D) Hypothalamic neurons secrete oxytocin and ADH that promute and inhibit the secretion and release‬
‭of hormones from the anterior lobe of pituitary gland.‬
‭E) Hypothalamic hormones also accumulate in nerve endings near the median eminence and‬
‭infundibulum and release into the capillary bed of the hypothalamohypophysial portal system for‬
‭transport to the pars distalis of the pituitary gland.‬
1- Ureter

2- Prostate
3- Vas deferens

4- Levator ani muscle


5- Left testicular vein
6- Roundligament( ligamentum teres hepatis)
7- Broad ligament
8- Ischiocavernous muscle

9- Bulb of vestibule (vestibular bulb)


Not: Round ligament hem 3 hem 4. komitede farklı yapıların isimleri olarak geçiyor. Her iki komiteye de her iki round
ligament yapısının görselini ekledim.

Prepared by
4th Committee Pratic/ Histology

Peg cells What is the phase of endometrium? Secretory

Tunica albugenia

Parathyroid Macula densa


Pars nervosa

Which one is inactive mammary


gland?b

What is these cells and their hormones?


Acidophils/ PRL and GH
What is these cells and their hormones? Parafollicular cells and hormone is calcitonin

What is the name of the cell? Umbrella cells

What is this epithelium type?


Transitional epithelium (in ureter)
I

Which one is vas deferens: A

g
Cell types about sperm development:
Seconder spermatocytes ?
Type B spermatogonium ?
(not sure of the answers)

Leydig cells
4th Committee Pratic/ Anatomy

1- Ureter

2- Vas deferens
ureter
tea artery
Mas deferens
3- Levator ani muscle

4- Prostate
5- Left testicular vein

6- Round Ligament

roundl gament

1
y
Broad
uterus
l gament
7- Ischiocavernosus muscle

8- Vestibular bulb
Committee 4 - Gathered Questions
2020 - 2021

Prepared by Tuğçe Özkan

Anatomy

1. Which artery is not crossed by ductus deferens?

A. Inferior epigastric artery


B. Testicular artery
C. Obturator artery
D. External iliac artery
E. Inferior rectal artery

2. Which of the following is not a cortical structure?

A. Vasa recta
B. Juxtaglomerular apparatus
C. Bowman’s capsule
D. Interlobular artery
E. Glomerule

3. Which of the following is in the superficial perineal pouch?

A. Deep transverse perineal muscle


B. Membranous part of the urethra
C. Bulbourethral gland
D. Superficial transverse perineal muscle
E. External urethral sphincter

4. Which muscle forms the lateral wall of the Alcock’s canal?

A. Obturator internus muscle


B. Levator ani muscle
C. Gluteus maximus muscle
D. Coccygeus muscle
E. Obturator externus muscle
5. Which structure does not contribute to the anterior wall of the inguinal canal?

A. Skin
B. Internal oblique muscle
C. Transversalis fascia
D. Superficial fascia
E. External oblique muscle

6. Which is not found in the inguinal canal?

A. Spermatic cord in men


B. Testicular artery
C. Ductus deferens
D. Round ligament in women
E. Ovarian artery

7. Which artery does not supply the breast?

A. Thoracoacromial artery
B. Internal thoracic artery
C. Inferior epigastric artery
D. Lateral thoracic artery
E. 3rd intercostal artery

8. Which of the following statements is false?

A. The longest part of the male urethra is the spongy part


B. Female urethra has a pelvic part and a perineal part
C. Membranous part of the male urethra is surrounded by external urethral sphincter
D. Perineal part of the female urethra is surrounded by external urethral sphincter
E. Preprostatic part of the male urethra is surrounded by the internal urethral sphincter

9. Left testicular vein drains into which vein?

A. Internal iliac vein


B. Prostatic venous plexus
C. Internal pudendal vein
D. Left renal vein
E. Inferior vena cava
10. Secretion from seminal vesicle drains into which duct?

A. Excretory duct
B. Tail of epididymis
C. Funicular part of ductus deferens
D. Ampulla of ductus deferens
E. Spongy part of the urethra

11. Which ligament limits the rectouterine pouch laterally?

A. Broad ligament
B. Suspensory ligament of ovary
C. Uterosacral ligament
D. Proper ovarian ligament
E. Round ligament

12. Which of the following is not in the vestibule of the vagina?

A. External urethral opening


B. Openings of small vestibular glands
C. Openings of Bartholin glands
D. Posterior fornix
E. Hymen

Physiology

1. Which of the following must occur for a pregnant woman to maintain a healthy
pregnancy?

A. The pituitary gland must secrete prolactin to sustain the placenta


B. The pituitary gland must secrete hCG to maintain the corpus luteum
C. The placenta must secrete FSH to maintain ovarian function
D. The placenta must secrete LH to maintain ovarian function
E. The corpus luteum must secrete progesterone to sustain the endometrium

2. Which one of the following classes of diuretics has the most powerful action?

A. Sodium channel blockers (triamterene, amiloride)


B. Loop diuretics (furosemide, bumetanide, ethacrynic acid)
C. Aldosterone antagonists (spironolactone, eplerenone)
D. Carbonic anhydrase inhibitors (acetazolamide)
E. Thiazide diuretics (hydrochlorothiazide, chlorthalidone)
3. A 65-year-old woman was taken to the emergency department after fracturing her
hip when stepping off a high curb. X-rays show a fracture at the head of the femur and
also reveal very low bone density in general. Which of the following hormonal profiles
for the pituitary-gonadal axis is predicted in this patient?
[FSH] [LH] [Estrogen]

A. Low Low Low


B. High High Low
C. Low Low High
D. High High High
E. Low High High

4. Which one of the following conditions cannot be considered among the causes of
prerenal acute kidney injury?

A. Malignant hypertension
B. Hemorrhage (trauma, surgery, postpartum, gastrointestinal)
C. Renal artery stenosis, embolism, or thrombosis of the renal artery or vein
D. Diarrhea or vomiting
E. Cardiac Failure, Myocardial infarction, or Valvular damage

5. A resident in internal medicine was called to the hospital room of an 85-year-old


patient in the middle of the night. The man was sitting up in bed coughing and was
severely short of breath. Crackles heard in both lungs suggested pulmonary edema.
Which diuretic is most appropriate for this patient?

A. Carbonic anhydrase inhibitor


B. Osmotic diuretic
C. Thiazide diuretic
D. Potassium-sparing diuretic
E. Loop diuretic

6. A 58-year-old man complained of loss of libido and reduced muscle strength.


Serum analysis showed reduced levels of testosterone. Reduced activity in which of
the following hormone-target cell axes could account for his symptoms?

A. FSH – Sertoli cell


B. LH – Sertoli cell
C. LH – spermatogonium
D. Luteinizing hormone (LH) – Leydig cell
E. Follicle-stimulating hormone (FSH) – Leydig cell
7. A 54-year-old woman received a life-saving kidney transplant 6 months ago and had
been well until the past few days. She now reports severe fatigue and dizziness upon
standing. Urinalysis is positive for glucose, and there is excessive excretion of HCO3
− and phosphate. In which segment of the nephron is the function most likely to be
abnormal?

A. Early distal tubule


B. Collecting duct
C. Proximal tubule
D. Late distal tubule
E. Loop of Henle

8. Which one of the following statements regarding glomerular filtration rate (GFR) is
correct?

A. Increased glomerular capillary hydrostatic pressure decreases GFR


B. Increased glomerular capillary colloid osmotic pressure increases GFR
C. Increased glomerular capillary filtration coefficiency decreases GFR
D. Increased Bowman’s Capsule hydrostatic pressure increases GFR
E. Any kind of obstruction of the urinary tract decreases GFR

9. Which one of the following organs and tissues is insulin-dependent?

A. Macrophages
B. Red blood cells
C. Skeletal muscle
D. Kidney medulla
E. Brain

10. A 4-year-old boy was diagnosed with visual disturbances due to a pituitary tumor
secreting excess growth hormone. Which of the following conditions would this boy
most likely develop without treatment?

A. Gigantism directly due to excess growth hormone


B. Dwarfism due to excess IGF-1 production
C. Dwarfism directly due to excess growth hormone
D. Acromegaly directly due to excess growth hormone
E. Gigantism due to excess IGF-1 production
11. What is the main source of ammonia produced by the kidneys?

A. Alanine
B. Leucine
C. Tryptophan
D. Glutamine
E. Glycine

12. Insulin and thyroxine arrive at an organ at the same time. Thyroxine causes an
effect on the organ but insulin does not. What could be the reason for this event?

A. Insulin is a lipid-soluble hormone and thyroxine is not


B. The organ’s cells have receptors for thyroxine but not for insulin
C. The target cells in the organ have up-regulated for thyroxine
D. Thyroxine inhibits the action of insulin
E. Thyroxine is a local hormone and insulin is a circulating hormone

13. Tolvaptan is the first orally active, specific V2 (vasopressin) receptor antagonist to
be licensed for use in humans. Which one of the following actions is not seen under
the influence of this drug?

A. It counteracts the actions of vasopressin by blocking the V2 receptor


B. Causes a decrease in serum sodium
C. It decreases the expression of the aquaporin channels
D. It causes an increase in free water clearance
E. Causes a decrease in urine osmolality

14. A healthy 25-year-old woman was a subject in an approved research study. Her
average urinary urea excretion rate was 12 mg/min, measured over a 24-hour period.
Her average plasma urea concentration during the same period was 0.25 mg/mL. What
is her calculated urea clearance?

A. 48 mL/min
B. 3 mL/min
C. 0.25 mL/min
D. 148 mL/min
E. 288 mL/min
15. Which one of the following effects is not observed by the actions of estrogen?

A. Inhibit osteoblastic activity in the bones


B. Cause cellular proliferation and growth of the tissues of the sex organs
C. Increase body metabolism and fat deposition
D. Promote the preservation of a homeostatic (eubiotic) vaginal microenvironment and
prevents infections
E. Cause the skin to become more vascular

16. Which one of the following can not be regarded among the causes of insulin
resistance?

A. Obesity/overweight (especially excess visceral adiposity)


B. Pregnancy, gestational diabetes
C. Lipodystrophy (acquired or genetic; associated with lipid accumulation in the liver)
D. Deficiency of growth hormone
E. Autoantibodies to the insulin receptor

17. Which one of the following statements regarding the obligatory urine volume is
not true?

A. The minimal volume of urine that must be excreted can be calculated as 600
mOsm/day / 1200 mOsm/L
B. Water loss from the skin, respiratory tract, and gastrointestinal tract would not
interfere with water balance during minimally obligatory urine volume disposal
C. The person drinking only seawater would have a higher obligatory urine volume
D. A normal 70-kilogram human must excrete about 600 milliosmoles of solutes each
day
E. Minimally obligatory urine volume disposal will ultimately cause dehydration when
water is not available to drink

19. Which of the following is not expected after a large hemorrhage?

A. Increased osmolar excretion


B. Increased aldosterone secretion
C. Increased sympathetic discharge
D. Decreased urine flow
E. Increased ADH secretion
20. What is the class of adrenal gland hormones that provides resistance to stress,
produce anti-inflammatory effects, and promote normal metabolism to ensure
adequate quantities of ATP?

A. Androgens
B. Gonadocorticoids
C. Mineralocorticoids
D. Glucocorticoids
E. Catecholamines

21. Which one is a correct explanation for the anti-inflammatory effect of cortisol
treatment?

A. Activation of phospholipase A2
B. Increased capillary membrane permeability
C. Increased formation of leukotrienes
D. Increased release of pyrogen from granulocytes
E. Stabilization of cellular lysosomal membranes

22. What is the normal human glomerular filtration rate (GFR) (mL/min)
approximately?

A. 75
B. 125
C. 25
D. 50
E. 225

23. Which one of the following is not an effect of cortisol?

A. Cortisol increases the permeability of the capillaries and allows protective fluid
leakage through interstitial compartments
B. Cortisol suppresses the immune system, causing lymphocyte reproduction to
decrease markedly
C. Cortisol decreases both migrations of white blood cells into the inflamed area and
phagocytosis of the damaged cells
D. Administration of large doses of cortisol causes significant atrophy of lymphoid tissue
throughout the body
E. Cortisol attenuates fever mainly because it reduces the release of interleukin-1 from
white blood cells
24. Why do you think that a person consuming high protein will have increased urine
concentrating ability?

A. High urea production helps hyperosmotic medullary interstitium


B. Increased glomerular filtration rate
C. Provides much more energy source for the kidney function
D. High structural protein formation in kidney tissue
E. The improved tubular function just because of high protein formation

25. Which one of the following does cause an increase in the secretion of growth
hormone?

A. Hyperglycemia
B. Somatostatin
C. Free fatty acid
D. Hypothermia
E. Exercise

26. Which of the following is not expected to cause vasoconstriction in the kidneys?

A. Sympathoadrenal system
B. ADH (or "Arginine Vasopressin Hormone"-AVP)
C. Renin-Angiotensin system
D. Aldosterone
E. Prostaglandins

27. Which of the following statements regarding thyroid hormone metabolism is not
true?

A. When the synthesis of thyroid hormone stops, the physiological effects of this event
would be observed within 2-3 days
B. In children with hypothyroidism, the rate of growth is greatly retarded
C. Thyroid hormones have a slow onset and long duration of action
D. Most of the thyroxine secreted by the thyroid is converted to triiodothyronine
E. They combine primarily with thyroxine-binding globulin and with thyroxine-binding
prealbumin and albumin

28. Which one is a result of cortisol administration to a patient with adrenal


insufficiency?

A. Increase insulin sensitivity in muscle


B. Increase corticotropin-releasing hormone secretion
C. Increase gluconeogenesis
D. Enhance wound healing
E. Increase ACTH secretion
29. The syndrome of inappropriate antidiuretic hormone secretion (SIADH) is caused
by the excess release of ADH. Which one of the following is increased because of
SIADH?

A. Plasma osmolality
B. Plasma oncotic pressure
C. Intracellular volume
D. Urinary flow
E. The concentration of plasma sodium

30. Which one of the following seems to be the main function of the prostate gland?

A. Production of glycogen
B. Secretion of alkaline mucus
C. Secretion of fructose
D. Maturation of sperm cells
E. Spermatogenesis

31. Which one of the following nephron segments and function pairs is correct in
respect of their functional relationship?

A. Proximal Tubule - Water is absorbed into the medulla


B. Thick Ascending Loop of Henle - Osmolarity of the fluid depends on the level of ANP
C. Descending Loop of Henle - About 2/3 of the electrolytes and water are reabsorbed
D. Early Distal Tubule - Most diluted place of the filtrate normally
E. Late DT and C. Collecting Tubules - Active electrolyte transport into the medullary
interstitium

32. A 28-year-old woman suffered severe trauma to the head in a horseback riding
accident that resulted in the complete transection of the pituitary stalk. The plasma
levels of which hormone would be expected to increase as a result of this accident?

A. Thyroid-stimulating hormone
B. Luteinizing hormone
C. Prolactin
D. Growth hormone
E. Oxytocin
33. Which of the following does not contribute to the formation of maximally
concentrated urine?

A. Presence of urea in the inner medullary interstitium


B. Active NaCl transport in the proximal convoluted tubule
C. Impermeability of the thick ascending limb of the loop of Henle to water
D. High water permeability of the collecting duct due to the presence of ADH
E. Active NaCl transport in the thick ascending limb of the loop of Henle

34. Which one of the following factors does not cause a shift of potassium into cells?

A. Very heavy exercise


B. Insulin
C. Beta-adrenergic stimulation
D. Alkalosis
E. Aldosterone

35. Which structure contains lysosomal-like enzymes?

A. Sertoli cells
B. Acrosome
C. Sperm tail
D. Sperm head
E. Centriole

[extra]36. Radiation treatment for a pituitary tumor in an 8-year-old boy resulted in the
complete loss of pituitary function. Which one of the following is he going to
experience as a result?

A. Increased ACTH
B. Hypothyroidism and goiter
C. Accelerated growth spurts
D. Increased TSH levels
E. Absent sexual maturation
Histology

1. Which histological layers of the endometrium constitute the functional layer and
disintegrate and shed at menstruation?

A. Compact layer and spongy layer


B. Basal layer and myometrium
C. Spongy layer and basal layer
D. Compact layer and basal layer
E. Basal layer, spongy layer, and compact layer

2. What are the anterior and posterior cloacal partitions divided by the urorectal
septum?

A. An anterior phallic part and a posterior pelvic part


B. An anterior urogenital sinus and a posterior anorectal canal
C. An anterior anorectal canal and a posterior urogenital sinus
D. An anterior pelvic part and a posterior vesical part
E. An anterior vesical part and a posterior phallic part

3. Which one of the following statements is false for the development of the female
reproductive system?

A. Development of the sex-specific internal ducts and external genitalia defined during
fetal development
B. Gonad differentiation begins at the 7th week of gestation
C. Chromosomal sex is determined at conception
D. Secondary sex characteristics defined at puberty
E. Female genital organ development is dependent on hormones and inhibitory
peptides

4. Which of the following statements is true?

A. Epithelial lining of the lactiferous sinuses is stratified cuboidal epithelium


B. Epithelial lining of the alveolar units is pseudostratified columnar epithelium
C. Mammary glands are simple alveolar glands
D. Lipid content of the colostrum is higher than the normal breast milk
E. Prolactin induces the myoepithelial cells to contract
5. Which one of the following is false for beta cells of Langerhans islets?

A. They stain red with the Mallory Azan method


B. They constitute about 70% of total islet cells
C. They contain numerous secretory granules about 300 nm in diameter and secrete
insulin
D. They have a dense polyhedral core and a pale matrix
E. They are generally located in the central portion of islets

6. Which one of the following is false for Thyroid follicles?

A. Follicles exhibit slightly basophilic cytoplasm with spherical nuclei containing one or
more prominent nucleoli
B. In routine H&E preparations, they are pale staining and occur as solitary cells or
small cluster of cells
C. Follicles reveal organelles commonly associated with both secretory and absorptive
cells (short microvilli on the apical surface) (numerous RER)
D. Follicles contain abundant lysosomes and endocytic vesicles identified as colloidal
resorption droplets
E. Follicles are responsible for the production of the thyroid hormones T4 and T3

7. Which of the following is wrong about the male reproductive system?

A. Vas deferens is an intratesticular duct that has the thickest muscular layer
B. Vesicula seminalis produces a fructose-rich secretion
C. Stereocilia is an apical surface modification that can be seen at epididymis epithelium
D. Leydig cells start to secrete testosterone at intrauterine 8th week and they can
increase in number after puberty
E. Compound tubuloalveolar glands of the prostate are under the control of
dihydrotestosterone

8. Which of the following cells are not expected to be seen in a 4-years old boy’s
testicular biopsy?

A. Myoid cell
B. Stereociliated columnar epithelium cell
C. Sertoli cell
D. Leydig Cell
E. Type A Spermatogonium
9. Which one is true for oxyphil cells of parathyroid glands?

A. They possess more abundant mitochondria than the chief cells


B. They are more abundant and smaller than the chief cells
C. They constitute a major portion of the parenchymal cells and have a secretory
function
D. They are more deeply stained with eosin than the chief cells
E. They are responsible for the secretion of PTH

10. Which one of the following statements is false for the zona fasciculata layer of the
adrenal cortex?

A. The zone contains sinusoidal capillaries arranged longitudinally between the columns
of parenchymal cells
B. The cells have spherical mitochondria with tubular and vesicular cristae, extensive
networks of SER
C. The cells are large epithelioid cells and they contain granules that stain intensely with
PAS reagent
D. The cells have many lipid droplets in their cytoplasm and are called spongiocytes
E. The polyhedral cells are larger than the cells of the zona glomerulosa and are
arranged in radial columns

11. Which of the following does not develop from the mesonephric duct?

A. Tubuli seminiferi recti


B. Caput part of epididymis
C. Vesicula seminalis
D. Ductus deferens
E. Ductus efferente

12. Which of the following does not have a role in male reproductive system
development?

A. Urethral bud
B. Yolk sac
C. Paramesonephric Ducts
D. Wolffian Ducts
E. Genital swelling
13. Which of the following is not a unit of renal corpuscle?

A. Juxtaglomerular cells
B. Parietal layer of Bowman’s capsule
C. Glomerular capillaries
D. Mesangium
E. Visceral layer of Bowman’s capsule

14. Which are the following are not the components of the filtration barrier?

A. Nephrin protein
B. Endothelium of glomerular capillary
C. Pedicels
D. Basal lamina
E. Podocyte cell body

15. Which one of the following statements is false for pituicytes of neurohypophysis?

A. Pituicytes occupy about 25% of the volume of the pars nervosa


B. Neurosecretory vesicles accumulate in these cells
C. Pituicytes contain lipid droplets, some pigments, and intermediate filaments
D. They are similar to neuroglial cells and function in supporting the axons of the pars
nervosa
E. These cells stain positively for glial fibrillary acid protein (GFAP)

16. Which of the following is not secreted from Sertoli cells?

A. Follicle-stimulating Hormone (FSH)


B. Mullerian inhibiting substrate (MIS)
C. Testicular plasminogen activator (TPA)
D. Androgen-binding protein (ABP)
E. Inhibin

17. Which cells are slightly basophilic, have spherical nuclei with a single prominent
nucleolus, and have small secretory vesicles producing melatonin?

A. Pinealocytes
B. Interstitial cells
C. Pituicytes
D. Oxyphil cells
E. Clear cells (C-cells)
18. Which one of the following organ(s) is/are not the derivatives of paramesonephric
ducts in the female reproductive system?

A. Ovary
B. Vagina ( upper portion)
C. Uterus
D. Tuba uterina
E. Cervix

19. Which hormone of the adenohypophysis is not secreted from basophilic cells?

A. Luteinizing hormone (LH)


B. Somatotropic hormone (GH - Growth hormone)
C. Follicle stimulating hormone (FSH)
D. Thyroid stimulating hormone (TSH)
E. Adrenocorticotropic hormone (ACTH)

20. In an ultrasonographic examination of a pregnant woman, oligohydramnios and


fetal unilateral renal agenesis are detected. Which is the most important cause of
renal anomalies?

A. Unilateral polycystic syndrome


B. Absence of ureteric bud
C. Oligohydramnios
D. Degeneration of mesonephros
E. Ureter duplication

21. At which stage of follicular development a deeply staining acidophilic layer called
zona pellucida appears around the oocyte and a sheet of connective tissue cells
known as Theca Follicle surrounds the follicle?

A. Primordial follicle
B. Secondary follicle
C. Graafian follicle
D. Antral follicle
E. Primary follicle
22. Which of the following is true for the development of the urinary system?

A. All three kidneys develop from the lateral mesoderm


B. Hox2 and Shh produced by the ureteric bud, cause the mesenchyme to epithelialize
for the preparation of excretory tubule differentiation.
C. The kidney initially localizes in the abdomen the later shifts into the pelvic region
D. Ureteric bud gives rise to the ureter, renal pelvis, and major-minor calyces
E. The collecting tubules of the kidney are derived from the metanephric mesoderm

23. Which of the following statements is true?

A. Lactiferous ducts are lined by simple cuboidal epithelium


B. Lipids contained in breast milk are secreted from the cell into the lumen by the
merocrine mode of exocytosis
C. During pregnancy, the end regions of the ductal structures in the mammary gland
develop by proliferation and form the alveoli structures responsible for secretion
D. In adult women, there are around 5-10 lobes in each mammary gland
E. Myoepithelial cells contract by the oxytocin stimulation secreted from the anterior
pituitary gland

24. Which of the following is not true for kidneys?

A. Mesangial cells participate indirectly in the glomerular filtration process by providing


mechanical support for the glomerular capillaries
B. The macula densa, a region of the initial portion of the distal convoluted tubule and
sensitive to changes in NaCl concentration and affects renin release by
juxtaglomerular cells
C. Principal cell of collecting tubules has the receptors of aldosterone which stimulated
the reabsorption of Na+
D. Cuboidal cells in distal convoluted tubules are shorter than those in the proximal
convoluted tubules and lack a prominent brush border
E. The Fanconi syndrome is a renal hereditary disease in which distal convoluted
tubules fail to reabsorb amino acids and glucose

25. Which of the following is wrong about male reproductive system?

A. Vas deferens is an intratesticular duct which has the thickest muscular layer
B. Vesicula seminalis produces fructose-rich secretion
C. Stereocilium is an apical surface modification which can be seen at epididymis
epithelium
D. Leydig cells start to secrete testosterone at intrauterin 8th week and they can
increase in number after puberty
E. Compound tubuloalveolar glands of prostate are under control of dihydrotestosterone
Biochemistry

1. Which of the following is the biologically active form of vitamin D?

A. Ergocalciferol
B. 24,25 dihydroxyvitamin D
C. 1,25 dihydroxyvitamin D
D. 25 hydroxyvitamin D
E. 1,24 dihydroxyvitamin D

2. Which of the following gastrointestinal hormones is a member of the secretin family


of GI hormones?

A. Neurotensin
B. Cholecystokinin
C. Pancreatic polypeptide
D. Somatostatin
E. Vasoactive intestinal polypeptide

3. Which of the following sentences are correct regarding calcium and phosphate
metabolism?

I. An increase in FGF-23 levels in blood causes a decrease in Vit D secretion


II. A decrease in PTH levels in blood causes an increase in Vit D secretion
III. Increased serum FGF-23 decreases Na/Pi cotransporter in tubular cells of the
kidney
IV. Vit D inhibits FGF-23 secretion from osteocytes
V. Increased serum PTH decreases Na/Pi co-transporters in tubular cells of the
kidney

A. I, II, III
B. I, III, V
C. III, IV, V
D. II, III, IV
E. III, I, IV

4. Which one of the following functions does not typify either prostacyclin (PGI2) or
thromboxane A2 (TXA2)?

A. PGI2 contracts coronary arteries


B. PGI2 dilates coronary arteries
C. PGI2 antagonizes platelet aggregation
D. TXA2 promotes platelet aggregation
E. TXA2 contract coronary arteries
5. Which of the following statements about the TSH hormone and its activity is not
correct?

A. It uses cAMP- second messenger system for its activity in the cells
B. It declines the endocytosis rate of colloid
C. It is accepted as the first-line screening test for the diagnosis of the thyroid
abnormalities
D. It increases thyroglobulin synthesis
E. It controls iodide uptake into the thyroid gland

6. Which of the following does not stimulate aldosterone secretion?

A. Hyponatremia
B. High blood potassium levels
C. Antidiuretic (ADH) hormone
D. Hypotension
E. Low blood volume

7. Which cofactor is used in the first reaction of catecholamine synthesis and works
together with tyrosine hydroxylase?

A. Tetrahydrobiopterin
B. S-Adenosylmethionine
C. Tetrahydrofolic acid
D. Pyridoxal phosphate
E. Flavin adenine dinucleotide

8. Which of the following statements is correct?

A. Adiponectin accelerates the accumulation of LDL in the arteries and increases the
risk of heart diseases
B. Resistin concentrations are low in obesity and insulin resistance
C. Glucagon hormone consists of 2 amino acid chains (α and β chains), two disulfide
bonds connecting the two chains, and a third disulfide bond within the α chain
D. Ghrelin’s task is to reduce appetite and to create a feeling of satiety after a meal
E. After the leptin hormone is synthesized from adipose tissue, it is released into the
blood and shows its effect in the hypothalamus
9. Which receptor works together with the thyroid hormone receptor in the nucleus of
the cell?

A. Insulin receptor
B. FGF23 receptor
C. Progesterone receptor
D. Retinoid X receptor
E. Peroxisome proliferator-activated receptor

10. Which enzyme catalyzes the coupling reactions in the thyroid gland for thyroid
hormone synthesis?

A. Catalase
B. Dismutase
C. Reductase
D. Oxidase
E. Peroxidase

11. Which of the following has 19 carbons?

A. Cortisol
B. DHEA
C. Pregnenolone
D. Cholesterol
E. Progesteron

12. Which of the following is not one of the chemical contents of the urine evaluated
in complete urinalysis (TIT)?

A. Glucose
B. Bilirubin-Urobilinogen
C. Protein
D. Density
E. Nitrite

13. Which does not belong to the proopiomelanocortin family?

A. B-lipotropin
B. Oxytocin
C. Endorphin
D. MSH (melanocyte stimulating hormone)
E. ACTH
14. During excretion of ammonium ions in the kidney, which intermediate of the TCA
(citric acid) cycle is formed?

A. Acetylglutamate
B. Oxaloacetate
C. Citric acid
D. α-Ketoglutarate
E. Succinyl-CoA

15. Which of the following steroidogenesis steps is common to adrenal glands, testes
and ovaries?

I. 17-OH Pregnenolone → DHEA


II. Testosteron → Estradiol
III. Pregnenolon → Progesteron
IV. Testosteron → Dihydrotestosteron

A. I, II
B. II, III
C. I, III, IV
D. I, III
E. I, II, IV

16. Tryptophan is best described by which of the following statements?

A. It is converted to thyroid hormones


B. It is a precursor of the pineal gland hormone melatonin
C. It is abundantly distributed in plasma proteins
D. It is an acidic amino acid
E. It is glucogenic only

17. Which of the following substances interact with the target cells by binding to a
receptor on the outer side of the plasma membrane?
I. 1,25 dihydroxycholecalciferol
II. Glucagon
III. Epinephrine
IV. Triiodothyronine

A. I and II
B. II, III, and IV
C. II and III
D. I, II, and III
E. III and IV
18. Which one of the following has the highest affinity for SHBG?

A. Pregnenolone
B. Cholesterol
C. Dihydrotestosterone (DHT)
D. Cortisol
E. Progesterone

19. Which is wrong for the microscopic properties of urine?

A. Urinary stones, cancers in the urinary system, infections in the urogenital system and
bleeding are the causes of hematuria
B. It is normal to have 5-6 bacteria in each microscope field
C. Having more than 5-6 erythrocytes in each area is hematuria
D. More than 5 leukocytes in each field are defined as pyuria
E. Abundant leukocytes are seen in urine in urinary tract infections

20. Which of the following sentences is not correct regarding calcium and phosphate
metabolism?

A. Major storage place of phosphate ion is bone tissue


B. Increase protein expression of Na/Pi II co-transporters in kidney results with
increased urinary phosphate excretion
C. Most of the phosphate absorption from intestines is independent of vitamin D
D. Calcium decreases phosphate absorption from intestines
E. Decreased serum FGF23 levels result with reduced urinary phosphate excretion

21. Which is not synthesized in the placenta?

A. hCG
B. Chorionic somatomammotropin
C. Cholesterol
D. Estriol
E. Progesterone

22. Which one of the following statements regarding steroid hormones is correct?

A. Cortisol released by the adrenal cortex regulates pituitary ACTH secretion with
positive feedback control
B. The synthesis and secretion of aldosterone is modulated by Na+ and K+
C. Vitamin D acts on mucosal cells of the small intestine by receptors on the outer side
of the plasma membrane
D. Cortisol increases the deposition of glycogen in liver by intracellular receptors
E. Steroid hormones act on targets by using secondary messengers
23. In which of the following acid-base disturbances anion gap evaluation is useful?

A. Metabolic alkalosis
B. Metabolic acidosis
C. Respiratory alkalosis
D. Respiratory acidosis
E. Uncompensated metabolic alkalosis

24. Which one is not a somatostatin effect?

A. Suppresses insulin secretion


B. Increases glucagon secretion
C. Reduces gastrin secretion
D. Reduces exocrine secretion of the pancreas
E. Inhibits secretion of growth hormone

25. Which one of the following has structural similarity with growth hormone?

A. Chorionic gonadotropin
B. Placental lactogen
C. TSH
D. LH
E. FSH

26. The patient is a 38-year-old female with an immune deficiency syndrome brought
to the emergency room with a fever of 39℃ and a three-month history of diarrhea.
What is the primary acid-base abnormality? pH: 7.33 PCO2: 27 mmHg (35-45 mmHg)
PO2: 90 mmHg (80-100 mmHg) Bicarbonate: 14 mEq/L (22-26 mEq/L)

A. Partially compensated respiratory acidosis


B. Fully compensated metabolic acidosis
C. Partially compensated metabolic alkalosis
D. Fully compensated respiratory acidosis
E. Partially compensated metabolic acidosis

27. Which of the following statements about metabolism in a fasting state is correct?

I. The dominant hormone is insulin


II. Acetyl CoA carboxylase enzyme activated
III. Hormone sensitive lipase enzyme activated
IV. Gluconeogenesis is activated
V. Glycogen synthesis is inhibited
A. I and II
B. I, II, III
C. IV and V
D. III, IV, V
E. I, II, V

28. Which of the following are not functions of intraglomerular mesangium?

A. Maintenance of renal architecture and production of erythropoietin


B. Regulating blood flow by their contractile activity
C. Providing mechanical support for the glomerular capillaries
D. Controlling the turnover of the glomerular basal lamina material by their phagocytic
activity
E. Secreting prostaglandins and endothelins

[extra]29. Which hormone shows its effect by the activation of adenylyl cyclase or
phospholipase C?

A. Glucagon
B. ADH
C. Oxytocin
D. TSH
E. Retinoic acid
Answer Key
Anatomy

1 2 3 4 5 6 7 8 9 10 11 12

B A D A D E C D D A C D

Physiology

1 2 3 4 5 6 7 8 9 10 11 12

E B B A E D C E C A D B

13 14 15 16 17 18 19 20 21 22 23 24

B A A D - - A - E B A A

25 26 27 28 29 30 31 32 33 34 35 36

E A A C C B E C C A B B

Histology

1 2 3 4 5 6 7 8 9 10 11 12 13

A B E A A B A E E C A C A

14 15 16 17 18 19 20 21 22 23 24 25

E B A A A B B E D C D A

Biochemistry

1 2 3 4 5 6 7 8 9 10 11 12 13 14

C E B A B C A A D E B D B D

15 16 17 18 19 20 21 22 23 24 25 26 27 28

A B C C B C C B B C B E D A
2019-2020 4th Committee Gathered Questions and Answers
Prepared By
Abdullah Yavuz
Berkay Yalçın
Beyza Şener
Büşra Arslan
Dilara Kısa
Rana Güngör
Umut Büyükarslan
BIOCHEMISTRY
1- Which hormone activates D-vit. to active form?

A) 24-hydroxylase

B) 25-hydroxylase

C) 1-a hydroxylase

D)24,25 hydroxylase

E) 1,25 hydroxylase

2- Which one doesn't belong to proopimelanocorticoin family?

A)ACTH

B)MSH

C)Endorphin

D)Oxytocin

E)B Lipotrophin

3- Which of the below statements us true for Tryptophan?

A) It is an acidic aminoacid.

B) It is the precursor of thyroid hormones.

C) It is the precursor of epiphysis hormone; melatonin.

D) It is glucogenic.

4-A 57 years old patient come to hospital for arterial blood gas analyazing. His routine
examination results were PH:7.37 his pCO2 63mmHg pO2 58mmHg HCO: 56 mEq/L

A) Partially compensated respiratory acidosis

B) Partially compensated metabolic acidosis

C) Fully compensated metabolic acidosis

D) Fully compensated respiratory acidosis

E) Fully compensated metabolic alkalosis

5-Which step of catecholamine production needs ascorbic acid?

A)Hydroxylation of Thyrosine

B)Decarboxylation of L-DOPA

C)Hydroxylation of Dopamine

D)Methylation of Norepinephrine

E)Hydroxylation of Phenylalanine
6-....... causes insulin resistance and ......... causes insulin sensitivity.

A)Resistin/adiponectin

B)Leptin/resistin

C)Ghrelin/Leptin

D)Leptin/Ghrelin

7- Which is wrong for somatostatin?

A) Decrease insulin secretion.

B) Increase glucagon secretion. C)

Inhibits pancreatic exocrine secretion.

D) Inhibits gastrin secretion.

E) Inhibits growth hormone.

8- Which one of the following is regarding to steroid hormone is correct?

A) Aldesteron synthesis and secretin is modulated by Na and K

B) Vit D is act on mucosal cell of small intestine by receptor on the outer surface of the plasma
membrane

C) Steroid hormones acts on target by secondary messenger

D) Cortisol secreted by adrenal gland regulates pituitary ACTH by positive feedback mechanism

E) Cortisol stimulates deposit of glycogen in liver by intracellular receptor

9- What is false about gastrin?

A) Binds to CCK-2 receptors activates Na/H transporters.

B) Gastrin inhibits histamin binding to parietal cell.

C) Gastrin stimulates histamin secretion from enterochromaffin cells.

D) Effects parietal cells by endocrine affect.

10- Which one of the followings increases appetite?

A)Cholecystokinin

B)Ghrelin

C)Pancreatic peptide

D)Somatostatin

E)Secretin
11- Which of the followings has highest affinity for SHBG?

A)Progesterone

B)Cortisol

C)Cholesterol

D)Pregnenolone

E)DHT

12- Ketonuria may be caused by all of the following except?

A)Bacterial infection

B)Vomiting

C)Starvation

D)Malnutrion

E)Diabetic acidosis

13- In which of the following acid-base disturbances, anion gap evaluation is useful?

A)Respiratory Acidosis

B)Respiratory Alkalosis

C)Metabolic Acidosis

D)Metabolic Alkalosis

E)Uncompansated Metabolic Alkalosis

14- Which of the followings are expected in an insulin-dominated metabolism?

I)Phosphoenolpyruvate carboxykinase is active.

II)Acetyl CoA carboxylase is inactive. III)Glycogen

phosphorylase is active.

IV)Hormone sensitive lipase is inactive. V)Fosfofructokinase

is active.

A) I and II

B) II and III

C) III and IV

D) IV and V

E) III and V
15- Which of the following has similar structure with growth hormone? A)Placental

lactogen

B)TSH

C)FSH

D)LH

E)Chorionic gonadotropin

16- Which one is activated by epinephrin's effect on alpha2 adrenergic receptors ?

A)Phospholipase C

B)cAMP

C)IP3

D)Ca

E)DAG

17- Which is the most important factor in fetal bone devolopment with sharing same
receptor for another hormone that regulate Ca metabolism?

A)PTH related protein

B)FGF23

C)Intact PTH

18- Which one is not synthesis in placenta?

A) hCG

B) Progesterone

C)Cortisol

D)Estriol

E)Chorinoic somatotropin

19- Which enzyme is only found in zona glomerulosa?

A) 21 beta hydroxylase

B) 18-hydroxysteroid dehydrogenase

C) 17 alfa hydroxylase

D) Aromatase

E) Steroidogenic acute regulatory (STAR) protein


20- Which one of the followings is not one of the TSH action?

A)Decrease endocytosis of colloid

21- Which of the sentences aren’t correct about regardings calcium and potassium
metabolism?

I) an increase in FGF23 level in blood cause a decrease in Vit D secretion

II) a decrease in PTH levels in blood causes an increase in VitD secretion

III) decreased serum FGF23 decreases Na/Pi cotransportes in kidney tubule.

22- K:17OH progesteron 1)3-beta-hsd

L:aldesteron 2)21-alfa-hydroxylase

M:DHEA 3)17-alfa-hydroxylase

In deficiency of which enzymes cause accummulation of which substance?

23- Guanine nucleotide bonding proteinin adenine cylcası active etmesiyle ilgili en iyi
tanım nedir?

24- PGI ve TXA-2 ile ilgili bir soruydu. Şıklar da bu ikisinin coronary arteri ve platelet
üzerine etkileri ile ilgiliydi ve yanlışı soruyordu.

25- Aşağıdakilerden hangisi/hangileri plazma membranındaki reseptöre bağlanarak


etki gösterir?

1- 1,25 dihidroksikolekalsiferol

2-Adrenalin

3- Glukagon

4- Triiyotironin

26- Aşağıdakilerin hangileri idrar testinde birlikte değerlendirilir?

1)protein-silendir

2)Protein-kristal

3bakteri-lökosit
4)eritrosit hemoglobin

Answer: 1,2, 3 ve 4

27- Adenilat siklazı veya fosfolipaz C yi aktive eden hormon hangisidir?

Answer: ADH

28- Bir insanda kan testi sonuçlarında serbest T4 miktarı normal ve artmış TSH
düzeyleri gözükmektedir buna göre aşağıdaki patolojik durumlardan hangisi olabilir?

Answer: subclinical hypothyroidism

29- Aşağıdaki moleküllerden hangisi nefronun proksimal kısmında sodyum geri


emilimini arttırır?

Answer: Angiotensin 2

30- Potasyum metabolizmasi ile ilgili hangisi yanlıştır?

Answer: Alkolosis causes potassium diffusion into circulation

31- Correct for leukocytes?

I-Round shaped

II-Have no granules

III-Seen in clusters

IV-bigger than RBC

Answer:1,3,4
HISTOLOGY

1. Which layer of the adrenal gland develop in the early years of life and constitute the
darkly staining acidophilic cells that are ranged in anastomosing cords?
A) Adrenal capsule
B) Zona glomerulosa
C) Zona fasiculata
D) Zona reticularis
E) Adrenal medulla
2. IN pregnant woman's ultrasonography in there are oligohydramnios and fetal
unilateral renal agenesis detected. What is the most important cause of renal anomaly?
A) No ureteric bud formation
B) Unilateral polycystic syndrome
C) Ureter duplication
D) Oligohydramnios
E) Degeneration of mesonephros

3. Which of the following wrong for male reproductive system?


A) Situation which having no semen is aspermia
B) Rise in environmental estrogen cause hypospadias
C) Cryptorchidism cause infertility
D) Leydig cell tumors cause gynecomastia in adult male
E) If the connection between periton and the processes vaginalis remains open, this will lead to torsion
of spermatic cord.

4. Which of the following does not join the filtration membrane?


A) Endothelium
B) Basal lamina
C) Podocyte cell body
D) Nephrin protein
E) Pedicel

5. Which functional layers is shed when pregnancy doesn’t occur or supply uterus when
pregnancy occurs?
A) Compact layer and basal membrane
B) Spongy layer and basal membrane
C) Compact layer and spongy layer
D) Compact, spongy, basal membrane
6.Which cell type gland correlation is true?
A) Pituocytes - Epifiz
B) Pinealocytes - Hypothalamus
C) Spongiocytes - Adrenal gland
D) Clear cell - Parathyroid
E) Oxyphil cell – Thyroid

7. Which one is wrong about female genital development?


A) Female genital organ development is dependent on hormones and inhibitory factors
B) Secondary sex character develops at puberty.
C) Genetic sex is determined at conception.
8. Which of the following does not have role of male reproduction system development?
A) Yolk sac
B) Paramesonephric ducts
C) Wolffian ducts
D) Genital swelling
E) Urethral Bud

9. Which one is true about urinary system development?


A) Collecting duct derived metanephric mesenchyme.
B) All 3 phase of kidney derived from Lateral Mesoderm
C) Ureteric bud gives rise to both calyx, pelvic part, and ureter.
D) Kidney is originally from abdomen and then descend to pelvic.
E) HoX2 and Shh gives signals for the development of mesonephric ducts.

10. Which one is false about hypothalamus histology?


A) It is in middle of base of brain, encapsulated ventral portion of 3rd ventricle
B) It coordinates endocrine system
C) It controls autonomic nervous system
D) In addition to ADH and oxytocin, hypothalamic neurons secrete steroids that promote and inhibit
secretion and release of hormones from anterior lobe of pituitary gland.
E) Nerve endings near median eminence and infundibulum gives hormones to capillary beds from
hypothalamohypophysial portal system these hormones go to anterior hypophysis

11. Which one is not true for mammary gland? A)


Mammary gland is tubuloalveolar gland.
B) Colostrum has higher protein than normal milk.
C) Prolactin activates myo-epithelial cells of mammary gland.
D) Alveolar is lined by simple cuboidal epithelium.
E) Lactiferous sinus lined by stratified cuboidal epithelium.
12. Which one of the following is not found in the renal corpuscle?
A) Parietal layer of Bowman’s capsule
B) Mesangial cells
C) Juxtaglomerular cells
D) Visceral layer of Bowman’s capsule
E) Glomerular capillary

13. Which one of following is false for thyroid follicular cells?


A) In routine H&E preparations, follicular cells pale staining and occur as solitary cells of small cluster
of cell.
B) Follicles reveal organelles commonly associated with both secretory and absorptive cells (short
microvilli on the apical surface) (numerous RER)
C) Follicular cells abundant lysosomes and endocytic vesicles, identified as colloid resorption droplets.
D) Follicular cells exhibit a slightly basophilic cytoplasm with spherical nuclei containing one or more
prominent nuclei.
E) Follicular cells are responsible production of T3 T4

14. What are the anterior and posterior cloacal portions divided by urogenital septum?
A) An anterior urogenital sinus and a posterior anorectal canal
B) An anterior anorectal canal and a posterior urogenital sinus
C) An anterior pelvic part and a posterior vesical part
D) An anterior phallic part and a posterior pelvic part
E) An anterior vesical part and a posterior phallic part

15. Which statement is wrong? A)


Epididymis has stereocilia.
B) Vas deferens is one of the intratesticular ducts.
C) Leydig cells start to secrete testosterone during 8th week of gestation.
D) Prostate gland works under/ depends on dihydrotestosterone.

16. Which one of the following cells seen at the 6 years old boy testicular biopsy?
A) Spermatid
B) Spermatozoon
C) Myeloid cells
D) Secondary spermatocyte
E) Primary spermatocyte

17. In which stage of the follicular development begins to form zona pellucida and theca
follicle?
A) Primordial follicle
B) Primary follicle
C) Secondary follicle
D) Antral follicle
E) Graffian follicle
18- Which one is not true for kidney?

A) Mesangial cell provide mechanical support.

B) Macula Densa sensitive to NaCl and mediate renin.

C) Cuboidal cells are shorter in DCT then PCT and DCT lack brush border.

D) Fanconi is a disease when DCT fail to absorb amino acid and glucose.

E) Aldosterone regulate Na reabsorbtion at collecting tubule.


19- 15-20%, eccenteric nuclei, basophils?

Answer: ACTH

20- Which of the followings not derived from paramesonephric duct?

A) Tuba uterina

B) uterus

C) cervix

D) upper part of vagina

E) vestibule

21- Wrong for alpha cells of pancreas?

A) Langerhans adacık hücrelerinin %70 ini oluşturur.

B) Genellikle adacığın merkezinde yerleşmişlerdir

C) 300 nm çapında insülin içeren çok sayıda salgı granülüne sahiptir

D) Yoğun polyhedral kor kısmı ve soluk boyanan matriks içerirler

E) Mallory’s Azanla kırmızı boyanırlar

22- Wrong for mesangial cells?

Answer: produce erythropoietin


1) Write the name of arrow labelled secreting cells? Peg Cells
2) Write the whole name of the area bordered with dashed line. Corpus Spongiosum

3) What is the name of the gland which indicated by arrow? Parathyroid gland

4) Nameof the structure indicated by arrow? Macula densa


5) What is the name structure indicated by arrow in hypophysis? Pars nervoza

6) Which of the pictures belongs to old seminal vesicule? A

7) Lactating mammary gland? A

A B

8) Which cell and which hormones secreted by these cells? Acidhophils; GH VE PRL
9) What is the name of the structures labelled with arrows at female reproductive
system? Primordial Follicules

10) Which organ? Vas deferens

11) What is the name of the layer shown with A? Myometrium


12) Corpus luteum? A

13) Type b spermatogonium


14) Which phase of uterus? Menstrual phase

ANATOMY

1. Which one of the following is not a cortical structure?

A) Glomerulus

B) vasa rectae

C) juxtaglomerular apparatus

D) interlobular artery

E) bowman capsule

2. Which artery does not supply breast?

A) 3rd intercostal

B) ınternal thoracic

C) Lateral thoracic

D) Thoracoacromial

E) inferior epigastric

3. Allock's canal lateral border is made of which muscle?

A) obturator internus muscle

B) obturator externus muscle

C) coccygeal

D) gluteus maximus muscle


4. Left testicular vein drains into which vein?

A) Inferior vena cava

B) Internal iliac vein

C) Right renal vein

D) Left renal vein

E) Femoral vein

5. Which structure doesnt pass through inguinal canal?

A) genitofemoral nerve

B) vas deferans

C) spermatid cord

D) testicular artery

E) ovarian artery

6. Which artery is not crossed with ductus deferens?

A) Inferior epigastric artery

B) Obturator artery

C) Testicular artery

D) İnferior rectal artery

7. Which ligament is lateral to rectouterine pouch?

A) broad ligament

B) proper ovarian ligament

C) uterosacral ligament

D) round ligament

E) suspensory ligament
8. Which of the following is wrong?

A) Female urethra has pelvic and perineal parts.

B) Membranous urethra of a male has the external urethral sphincter.

C) Longest part of male urethra is the spongy urethra.

D) Preprostatic urethra of a male has the internal urethral sphincter.

E) Perineal part of female urethra has the external urethral sphincter.

9. Which is not in the vestibule of vagina?

A) hymen

B) external uretral opening

C) opening of small vestibular glands

D) opening of bartholin gland

E) posterior fornix

10. Which one doesn’t found in deep perineal puch?

A) Deep transverse perineal muscle

B) Membranous part of urethra

C) External Urethral Sphincter D) Superficial transverse perineal muscle

E) Bulbourethral gland.

11. Ejaculatory duct formed by?

Answer: Excretory duct of seminal vesicle+ampulla of vas deferans


1- Left Iliohypogastric nerve
2- Ureter
3- Internal Urethral Orifice

4- Round Ligament

5- Ischiocavernosus muscle
6- Vas Deferens

7- Superficial Transverse Perineal Muscle


8- Bulb of Vestibule

PHYSIOLOGY
1. Which one result of cortisol?

A) Increase gluconeogenesis

B) Increase insulin sensitivity


C) Increase ACTH

D) Increase glucagon synthesis

2. Which one of the following factors does not cause a shift of potassium into cells?

A) Insulin

B) Aldosterone

C) Very heavy exercise

D) Beta adrenergic stimulation

E) Alkalosis

3. Tolvaptan is oral active (ADH) V2 antagonist. Which of the following is not true about
action of tolvaptan?

A) Conteract ADH by blocking v2

B) Decreases aquaporin channels

C) Increases free water clearance

D) Decreases urine osmolarity

E) Decreases sodium concentration in extracellular compartment

4. What is the most powerful diuretics?

A) Na channel blockers

B) Loop diuretics

C) Thiazide diuretics

D) Aldosteron antagonists

5. Which one is not vasoconstrictor?

A) ADH

B) Renin angiotensin system

C) Aldosterone

D) Symphatic system

E) Prostoglandin

6. Thyroxin and insulin are going to a cell. Thyroxin has an effect on cell but insulin
doesn't. What could be the reason for this? A) Target cell have upregulation for thyroxine.

B) Insuline is inhibited by thyroxine

C) thyroxine is local hormone and insuline is circulatory

D) Insuline is lipid-soluble
E) Cell has receptor for thyroxine but not for insuline.

7. Which tissue use glucose independent from insulin?

A) Brain

B) Adipose tissue

C) Uterine

D) Skeletal muscle

E) Cardiac muscle

8. Which one is a correct explanation for the anti-inflammatory effect of cortisol


treatment?

A) Increased capillary membrane permeability

B) Increased formation of leukotrienes

C) Stabilisation of cellular lysosomal membranes

D) Increased release of pyrogen from granulocyte

E) Activation of phospholipase A2

9. Sperm cannot fertilize an egg unless they have been in the female reproductive tract
for several hours. This enhanced ability of the sperm do fertilize the ovum is called
capacitation. In vitro studies indicate the sperm can fertilize ovum if they are first
washed before being introduced to the ovum. What is the suggestion of it?

A) Capacitation process involves release of some inhibiting substance and some substance from sperm.

B) Capacitation process do not depend temperature.

10. Which one is not contribute to form maximal concentration urine?

A) Na/Cl transport channel in proximal tube

B) Na/Cl transport channel in thick ascending loop of henle

C) Impermeable water of thick ascending loop of henle

D) Water permeability of collecting duct with ADH

E) Presence of urea in inner medullary

11. What is not expected in heavy hemorrhage?

A) Increase ADH

B) Increase sympathetic discharge

C) Increase aldosterone

D) Decrease urine flow

E) Increase osmolar excretion


12. Which of the following statements regarding obligatory urine volume is NOT true?

a) A normal 70-kg human must excrete about 600 mOsm of solutes each day.

b) Obligatory urine volume can be calculated as 600mOsm/day / 1200mOsm/L

c) A person who drinks a lot of sea water would have increased obligatory urine volume.

d) Minimal obligatory urine volume disposal will eventually cause dehydration when water is not
available to drink.

e) Water loss from skin, respiration, and GIT do not interfere with water balance during obligatory urine
volume disposal.

13. Which one of the following can not be regarded among the causes of insulin
resistance?

A) Obesity/overweight(especially excess visceral adiposity)

B) Deficiency of growth hormone

C) Autoantibodies to the insulin receptor

D) Pregnancy, gestational diabetes

E) Lipodystrophy (acquired or genetic associated with lipid accumulation liver)

14. Which of following is responsible for act of ejaculation?

A) Parasympathetic innervation

B) Stimulation of cowper gland

C) Rhytmical peristaltic movements

D) Contraction of urethra along alone

E) Contraction of prostate gland and urethra

15. Radiation treatment for a pitiutary tumor in an 8years old boy resulted in the
complete loss of pituitary function. Which one of the following is he going to experience
as a result?

A) Hypothyroidism and goiter

B) Absent sexual maturation

C) Increased ACTH

D) Increased tsh level

E) Accelerated growth spurts


16. Why do you think that a person consuming high protein will have increased urine
concentrating ability?

A) Increase GFR

B) Has more energy

C) Tubular function increase

D) High urea production helps hyperosmotic medullar intersititum

17. Which one is not a cause of prerenal acute failure?

A) Anesthesia

B) Malignant hypertension

C) Diarrhea

D) Sepsis

E) Renal artery stenosis, embolism

18. Which one is the adrenal hormone class that provides resistance to stress, anti-
inflammatory and promotes normal metabolism to ensure adequate ATP?

A) Glucocorticoids

B) Catecholamines

C) Mineralocorticoids

D) Androgens

E) Gonadocorticoids

19. Which of following is wrong about thyroid metabolism?

A) Thyroid hormones are carried by TBG(thyroid binding globulin),prealbumin and albümin.

B) If thyroid hormone secretion stop,the physiological effect would be observed within 2-3 days.

C) Mostly produced thyroxine is converted to triiodothyronine.

D) Thyroid hormone has slow onset and long duration of action.

E) In child hypothyroidism rate of growth retarded.

20. Which of the followings can be compensatory mechanism when a person who has
respiratory acidosis(his blood hydrogen concentration change 7.4 to.7.3)

A) Decrease bicarbonate concentration in plasma


B) Decrease ammonium concentration in urine

C) Stimulation of peripheral receptor

D) Inhibition of central receptor

21. Which situation increases GH secretion?

A) Exercise

B) Hypothermia

C) Free fatty acids

D) Hypothyroidism

22. Which one is used for production and secretion ammonium by proximal tubule
buffering of excess acid?

A) Glutamine

B) Histamine

C) Metionin

D) Tryptophan

E) Alanine

23. Which one is must occur for a healthy pregnancy?

A) Progesterone from corpus luteum

B) FSH from pituicyte

C) LH from pituicyte

D) hCG

24. Excessive androgen sentezi durumunda hangisi görülmez?

A) Yetişkin erkekte kolayca belli olur

25. What is the normal human glomerulus filtration rate (GFR) (mL/min)
approximately?

A) 25

B) 50

C) 75

D) 125

E) 225
26. Which one of the following is not an effect of cortisol?

A) Cortisol increases permeability of the capillaries and allow protective fluid leakage through interstitial
compartments.

B) Cortisol decreases both migration of white blood cells into the inflamed area and phagocytosis of
damaged cells.

C) Cortisol suppresses the immune system causing lymphocyte reproduction to decrease markedly.

D)Cortisol attenuates fever mainly because it reduces release of interleukin-1 from white blood cells.

E) Administration of large doses of cortisol causes significant artrophy of lymphoid tissue throughout
the body.

27. Which of the following increases when a cranial hemorrhage occurs?

1- ADH

2- Aldosterone

3- Plasma Sodium concentration

4- ANP

5- Plasma Osmolarity

28. The syndrome of unappropriate antidiuretic hormone secretion (SIADH) is caused


by the excess release of ADH. Which one of the following is increased because of
SIADH?

A) Intracellular volüme

B) concentration of plasma Na

C) Urinary flow

D) Plasma oncotic pressure

E) Plasma osmolarity
2018-2019 Committee 5 9- Which structure is 2cm lateral to supravaginal part of
-Anatomy- the cervix? A) Ovarian Artery
1- Which of the following pass through the Alcock canal? B) Urethra
A) Pudendal Nerve C) Ovary
B) Femoral Nerve D) Uterine Tube
C) Genitofemoral Nerve E) Ureter
D) Illioinguinal Nerve
E) Pelvic Plexus Nerves 11- In which part of male reproductive system the
spermatozoa synthesized?
2- Which one of the following is not in the urogenital A) Seminiferous tubules
triangle? A) Crus Penis
B) Anal Canal -Physiology-
C) Crus Clitoris 12- In which condition urinary flow is less than normal?
D) Vagina A) Diabetus Mellitus
E) External Urethral Orifice B) Diabetus Insipidus
C) Sympathetic Stimulation
3- Which of the following ligament interconnects the ovary D) Increased Renal Arterial Pressure
with the uterus? A) Round Lig E) Infusion of Mannitol
B) Suspensory Lig
C) Proper Lig 13- Which one of the following waste product is different?
D) Mesovarioum Lig A) Urea
E) Broad Lig B) Uric Acid
C) Bilirubin
4- What is in the kidney is not a cortical structure? D) Ammonia
A) Vasa Recta E) Creatinin

5- Which structure constricts ureter? 14- Which one of the following values will be above, normal
A) Common Iliac Artery in diabetic patient with a blood glucose concentration of
B) Internal Iliac Artery 700 mEq/L?
C) External Iliac Artery A) Alveolar CO2
D) Abdominal Aorta B) Urine Flow
E) Testicular Artery C) Arterial pH
D) Intracellular Volume
6- Which of the following is not found in inguinal canal? E) Plasma Na Concentration
A) Genital Br. of Genitofemoral
B) Ilioinguinal 15- Which of the following’s reabsorption rate is increased
C) Femoral N. by ADH from collecting tubule?
D) Ductus Deferens A) Urea
E) Testicular A. B) H
C) NH4
7- Where does the ovarian artery originate from? D) Na E) K
A) Common Iliac Artery
B) Internal Iliac Aretery 16- Which of the following features the renal corpuscle that
C) External Iliac Artery enhance its filtering capacity?
D) Abdominal Aorta 1. Large granular capillary surface area
2. Thick selectively permeable filtering membrane
8- Which structure is not in the urogenital triangle? 3. High capsular hydrostatic pressure
A) Major Vestibular Gland 4. High glomerular capillary pressure
B) Crus Penis 5. Mesangial cells regulating the filtering surface area
C) Bulb of Penis A) 1,2,3 B) 2,4,5 C) 1,4,5 D) 2,3,4 E) 2,3,5
D) Vestibular Bulb
E) Anal Canal 17- Which one of the following signs of renal failure is
caused by kidney? A) Edema
B) Anemia A) It is classified as titratable acid.
C) Acidosis C) Produced in epithelium of distal tubule
D) Hypertension D) Decrease plasma HCO3 level
E) Uremia E) Impermeable for proximal tubule

18- What percentage of the filtered load of sodium is 24- Which of these are true about male urinary system?
reabsorbed by the proximal tubule? A) Erection is innervated by sympathic nervous system
A) 15 B) 25 C) 45 D) 65 E) 95 B) Ejaculation is innervated by parasympathic system
with sacral plexus.
19- From which structure estrogen and progesterone are C) something about bulbourethral gland
secreted at last 7 months of pregnancy?
A) Ovum 25- What is the obligatory urine volume in one patient who
B) Anterior pituitary is suffering from renal disease and has an impaired urine
C) Posterior pituitary concentrating ability with a 400 mOsm/L of maximum urine
D) Uterus osmolarity? A) 0.5 L/day
E) Placenta B) 1.0 L/day
C) 1.5 L/day
20- Which one of the following statements regarding D) 2.0 L/ day
hypogonadism in male is not correct? E) 2.5 L/day
A) When a baby loses his testes before puberty
(anorchidism), he continue to have infantile sex organs and 26- Which one of the following does cause increase in Renin
other infantile sex characteristics throughout his life. release?
B) The high of an adult eunuch is smaller than of a A) Hypertension
normal man. B) Increased Aldosterone
C) In eunuch, bones are thin and muscles are weaker C) Hypovolemia
than those of a normal man. D) Increased Ang 2
D) In a castrated adult male, sexual desires may E) Renal Arterial Stenosis
decrease but may not be lost.
E) When a man is castrated after puberty, some of his 27- Which of the following is released by juxtaglomerular
male secondary sexual characteristics revert to those of a when NaCl concentration decreases?
child. A) ADH
B) Renin
21- Which one of the following conditions could C) Adenosine
be responsible for increased GFR? A) Afferent D) Aldosterone
arteriole resistance increased E) Angiotensinogen
B) Efferent arteriole resistance decreased
C) Plasma protein concentration decreased 28- Which of the followings is a result of SIADH, which is
D) Urine flow through the urethra is decreased caused by excess release of ADH?
E) Sym. nerve activity to kidney is increased A) Urinary Flow
B) Plasma Osmolarity
22- Which of the following hormone’s excess secretion C) Intracellular Volume
would cause hyponatremia? D) Plasma Oncotic Pressure
A) Vasopressin E) Concentration of Plasma Sodium
B) ANP
C) Norepinephrine 29- Which of these is true for implantation in uterine wall?
D) Insulin
E) Aldosterone 30- Which one of them does not cause infertility in females?
A) Inflammation in Fallopian tubes.
B) Excess HCG.
C) Endometriosis.
D) Reduced Gonadotropins Level.
E) No Ovulation.
23- Which one is true about Nh3?
31- Which of the following statements regarding acid base A) Proceeds formation of zona pellucida B)
disturbance is not true? Involved infiltration of the endometrium by the
A) Increased aldosterone levels will cause reabsorption of synsitiotrophoblast.
H+ from distal nephrons resulting in metabolic acidosis B) C) Occurs when embryo consist of approximately 128 eggs.
Metabolic alkalosis is because of loss of nonvolite or fixed D) Occurs 3-5 days after fertilization
ion from body E) Is inhibited by secretion of progesterone from
C) Persistent diarrhea will cause loss of bicarbonate ions corpusluteum.
frombody resulting in metabolic acidosis
D) Diabetes will be resulting in metabolic acidosis because of -Histology & Embryology-
accumulation of keto acid
E) Hyperventilation will cause respiratory alkalosis 39- In ultra sonographic examination oligohydramnios
and fetal unilateral renal agenesis are detected. What is the
32- Which one of the symptom is caused by the loss most important cause?
of a hormone which is produced by kidney? A) Edema B)
Anemia C) Acidosis D) Hypertension E) Uremia 40- Which of the following is not true for true
congenital renal malformation
33- What percentage of filtreted load of sodium is A) Unilateral renal agenesis is compatible with life
reabsorbed by the proximal tubule? and present with genital anomalies
A) 15 B) 25 C) 45 D) 65 E) 85 B) Urachal cyst results in urine from urachus C)
Bilateral renal agenesis is incompatible with life and
34- Which of the followings are true? associated with oligoamniosis
Micturition reflex is, D) Wilms tumor of kidney has mesodermal origin
1. Initiated by stretch receptors in ureter. andmetanephric tissue of kidney have failed to differentiate
2. Relies on para-sympathetic impulses from micturition into renal kidney tissue
center in S2 and S3. E) Pancake kidney is resulting from the fusion kidneys
3. Results in contraction of Detrusor Muscle. whilethey were in pelvis
4. Results in contraction of Internal Urethral Sphincter
Muscle. 41- Which is connect about derivation of genital tubercle?
5.İnhibits motor neurons of External Urethral Sphincter A) Endoderm , Somatic Mesoderm
Muscle. B) Ectoderm ,Somatic Mesoderm
C) Ectoderm, Intermediate Mesoderm
35- Which of the following is not a function of testosterone? D) Endoderm ,Intermediate Mesoderm
A) Spermatogenesis E)Endoderm , Ectoderm
B) Testosterone increases basal metabolism
C) Increases bone matrix 42- Which one is incorrect?
D) Increases protein formation and muscle development A) Clitoris-penis
E) Eliminates excess water and mineral from the body B) Bartholin gland-Cowper gland
C) Ovary-testis
36- Which one of the following is not observed after the D) Oogonia-spermatagonia
renal blood perfusion rate is reduced? A) Oligourea E) Labro minora-scrotum
B) Increased renin level
C) Increased aldosterone level 43- Which one of the following is not correct about
D) Increased excreted water amount structures and their developmental process? A) Female
E) Decreased excreted sodium amount Gonad: Primordial follicle-Primary FollicleSecondary Follicle-
Grafian Follicle- Corpus Albican-Corpus
37- Which of the following is most likely increase the GFR? Luteum
A) Volume Depletion B) Uterus: Menstruation- Proliferative Phase-Secretory
B) Ureter Blocking Phase
C) Cant Mesengial Contraction C) Male Gonad: Spermatogonia, Primary
D) Dilation of Afferent Arteriol SpermatocyteSecondary Spermatocyte-Spermatid-
E) Renin Secretion From Juxtaglomerular Spermatozoa
38- Which is involved in implantation of zygote in uterus? D) Mammary Gland: Immature Gland- Pregnancy Active
Gland-Regressing Gland
E) Female Sexual Development: Pre-puberty-
PubertyProductive Phase-Menopause

44- Which of the following is CORRECT about derivation of 50- Which follicular stage also called as antral follicle?
genital tubercle? A) Primordial Follicle
A) Endoderm and somatic mesoderm B) Primary Follicle
B) Ectoderm and somatic mesoderm C) Scondary Follicle
C) Ectoderm and intermediate mesoderm D) Mature Follicle
D) Endoderm and intermediate mesoderm E) Graffian Follicle
E) Endoderm and ectoderm
51-Which of the following is not unit of the renal corpuscle?
45- What is the name of the acidophilic membrane A) Visceral Layer of Bowman Capsule
surrounding the oocyte? B) Parietal Layer of Bowman Capsule
A) Grafian Follicle B) Theca Interna C) Zona Pellucida D) C) Mesangium
Antrum E) Corona Radiate D) Juxtaglomerular Cells
E) Glomerular Capillary
46- Which sequence is correct about sperm journey in
reproductive system? 52- Which one is correct?
Seminiferous Tubule, Straight Tubule, Rete Testis, Efferent A) Testis is primarily elaboration of mesonephros
Duct, Epididymis, Vas Deferans, Urethra B) Vagina derived from urogenital sinus(2/3) and
paramesonephric duct(1/3)
47- Which of the following is correct about C) Genital tubercle gives rise to vagina in female
reproductive developmental outcome when dht is missing?
A) The gonads develops into ovary , the duct into 53- Which of the following is wrong corresponding to
uterus and genital tubercle into the male external genitalia female and male genitalia?
B) The gonads develops into ovary, the duct into vas A) Clitoris for female and penis for male
deferans and genital tubercle into the male external B) Bartholin gland for female and bulbourethral gland for
genitalia C) The gonads develops into testis , the duct into male
uterus and genital tubercle into the male external genitalia C) Oogonia for female and spermatagonia for male D)
D) The gonads develops into Testis ,the duct into vas Ovum for female and testis for male E. Labia minora for
deferans and genital tubercle into female external genitalia female and scrotum for male
E) The gonads develops into testis, the duct into vas
deferans and genital tubercle into male external 54- What are the anterior and posterior cloacal portions
divided by urogenital septum?
48- Which one of the following is correct with respect A) An Anterior Urogenital Sinus and a Posterior Anorectal
to normal development of the gonadal-ductal- genital Canal
axis? A) Testis is primer on elaboration of the B) An Anterior Anorectal Canal and a Posterior Urogenital
mesonephron B) The mesonephric duct gives rise to Sinus
prostate and bulbourethral gland C) An Anterior Pelvic Part and a Posterior Vesical Part
C) The penile urethra in males is homologous with the D) An Anterior Phallic Part and a Posterior Pelvic Part E) An
vagina in females Anterior Vesical Part and a Posterior Phallic Part 55-
D) The vagina is derived from both urogenital sinus (2/3) and Which one is INCORRECT?
paramesonephric duct (1/3) A) Leydig cell produces testerogen(?)
E) The genital tubercle gives rise to the vagina in females. B) Sertoli cell is the “nurse” in testes that regulates germ
cell proliferation and differentiation.
49- About mammary glands, which one is correct? C) Myoid cell in the seminiferous tubule for help movement
A) Mammary glands are modified Serous glands of sperm
B) Mammary glands are modified Mucous glands D) Epididimis transports and stores sperm cells that are
C) Mammary glands are modified Sebacous glands produced in the testes.
D) Mammary glands are modified Sudoriferous glands E) Spermatogenesis begins at puberty, when testosterone
E) Mammary glands are modified Ceruminous glands levels rise.
56- In hyperosmolar hypernatremia, while sodium levels are low
which molecule increases serum osmolarity?
A) Glucose B) Alkaline phosphatase C) Creatine
D) Hemoglobin E) Phosphorus

57- What can we say patient acid base status detected increased
anion gap in a patient admitted emergency department?
A) Respiratory Acidosis
B) Respiratory Alkalosis
C) Metabolical Acidosis
D) Metabolical Alkalosis
E) Normal Acid Base Status

58- Idrarda eritrosit testi negatif cikmasina ragmen strip testte


hemoglobin pozitifse bu durum neden olur?
A-Contamination
B-non nitrite bacteria

59- Collecting a midstream, clean urine sample will prevent


contamination of:
A) Erythrocyte
B) WBC
C) WBC Casts
D) Simple squamous epithelium
E) Tubular Cell Epithelium

60- Which one of the following statements about aldosterone is


incorrect ?
A) Aldosterone is released in response to increase in ECF. B)
Aldosterone is released in response to decrease renal perfusion.
C) Aldosterone secretion increases the reabsorption of Cl.
D) Angiotensin 2 stimulates aldosterone secretion from the adrenal
cortex.
E) Aldosterone increases Na reabsorption from the distal tubules.
61- Which is the most found in bladdery and renal Stones?
A) Calcium Oxalate
B) Lystine
C) Triple phosphate
D) Uric Acid/Urates
E) Leucine

62- Relating to regulation of acid base incorrect one?


A) Increased renal glutaminase activity in acidosis is found B)
Buffering capacity of hb is greater than buffering capacity of plasma
amino acids
C) Most effective buffer (system in?) Physiological ph is
bicarbonate buffer system
D) Protein secretion and bicarbonate reuptake in renal tubule
ottur by the help of cerbonic anhydrase63-

-Biochemistry- -Anatomy Practical-


-Histology & Embryology Practical -
Öykü Erbil & M. Alperan Yavuz

You might also like